You are on page 1of 99

FINANCIAL PLANNING ACADEMY

Introduction to Financial Planning Part - 1


1. What are the common client characteristics for a high income earner? (a) Often debt laden for purchase of home (b) Very focused on post retirement income (c) Employed or self-employed. Taxation strategies more important (d) Starting to accumulate assets, often going into debt to achieve them 2. What is step two of the financial planning process? (a) Identification of financial problems (b) Goal setting (c) Preparation of written alternatives and recommendations (d) Data gathering 3.Which one of the following statements would be INAPPROPRIATE if included in a Statement of Advice for a client? (a) We expect that your investments will return at least 5% per annum on average over the next 5 years (b) Your investments will return at least 5% each year for the next 5 years (c) Investments in these asset classes have averaged 5% return each year. We would be expect that to continue for you (d) Investments in these asset classes returned 5% last year. We anticipate that this return will continue to be achieved over the long term 4. A data collection form should collect: (a) Both qualitative and quantitative information (b) Quantitative information only (c) Qualitative information only (d) Financial information only 5. Which of the following statements is INCORRECT? (a) Quantitative data is facts and figures (b) Qualitative data can be used to make inferences (c) A client's age is an example of qualitative data (d) All data can be divided into either quantitative or qualitative data 6. An example of QUALITATIVE data is: (a) Unit trust balance (b) Age (c) Preferred retirement date (d) Outstanding debts

Financial Planning Academy

7. If a client has an investment time frame of six years, what is the planning horizon? (a) Intermediate (b) Medium-term (c) Long-term (d) Short-term 8. Clients often have concerns about what is involved in seeing a financial adviser. Raising and discussing those concerns with the clients before they raise them themselves can have many advantages. These advantages include: I. It encourages openness in the discussion between the client and the adviser II. It gives the client confidence in the adviser, since potentially negative issues are being freely raised III. It allows for a quicker interview, thus saving time for all parties IV. It allows the adviser to take control of the interview (a) I and II (b) I and IV (c) II and III (d) I, II and III

9. Which of the following asset classes can produce both income as well as growth?
(a) Equities only (b) Property and equities (c) Property, equities, fixed interest, and cash (d) Property, equities, and fixed interest 10. With respect to 'risk/return trade off', what is generally traded away in order to achieve higher returns? (a) Lower investment returns (b) Asset allocation (c) Higher investment returns (d) Stability of income/growth 11. Which of the following is an example of liability risk? (a) Through your actions, causing injury to others, or damage or loss to others' property (b) Death (c) Loss or damage to one's own property (d) Loss of income over an extended period of time 12. Installing a car alarm is an example of: (a) Risk avoidance (b) Risk control (c) Risk transference (d) Risk retention

Financial Planning Academy

13. Which of the following is NOT a key component of a will? (a) Financial adviser details (b) Revocation clause (c) Appointment of an executor (d) Residuary clause 14. What is an advantage of having a separate account for known bills? (a) It allows the client to keep all their income together. This will reduce the client's spending and ensure that the bills can always be paid (b) It allows the client to ensure that the 'bill account' always has enough money in it to cover recurring expenses (c) It ensures that the client has enough money to spend on discretionary items (d) It allows the client to keep tabs on all the income they are receiving 15. A fixed interest investment is distinguished by two factors. They are: (a) The term and the interest rate are both variable at the start of the investment (b) The term and the interest rate are both fixed at the start of the investment (c) The term is fixed at the start of the investment, but the interest rate is variable (d) The interest rate is fixed at the start of the investment, but the term is variable 16. Which of the following is a DISADVANTAGE of managed investments? (a) Poor liquidity (b) The need for active management (c) Lack of diversity (d) Loss of personal control over decision making 17. When is an investment said to be negatively geared? (a) When the investment income is less than the cost of the investment (b) When the investment income is greater than the cost of the investment (c) When the investment income is the same as the cost of the investment (d) When the investment income is negative

18. Why is retirement planning becoming increasingly important?


(a) (b) (c) (d) Governments are less likely to want to support the elderly There are better medical treatments ensuring longevity Clients want to be able to afford expensive overseas holidays when they retire We are moving towards a self-funding retirement structure

Financial Planning Academy

19. An adviser is reviewing a client's situation. What might be a practical benefit to the adviser of conducting legislative research? I. The adviser might amend the investment strategy in light of an announced change to tax law II. The adviser might suggest the client retain certain investments in light of changes to their tax treatment III. The adviser might suggest that implementation of a recommendation be delayed in order to take advantage of a new rule IV. The adviser might suggest that implementation of a recommendation be brought forward in order to take advantage of a rule before it changes (a) I, II, III and IV (b) I, II and III (c) I, III and IV (d) II, III and IV 20. An investment product has experienced a loss of customers. A QUALITATIVE reason for this might be: (a) The return on that asset class is lower than other asset classes. Consequently, investors are moving out of this product in search of better returns (b) The tax treatment of the product has changed, making it less attractive than it was previously (c) A new product has been introduced that is likely to be a better investment for clients (d) The fund's original management team has left and their replacement team is not as well respected. Consequently, customers are leaving 21.What are the four phases of the business cycle? (a) Recovery, expansion, contraction, and recession (b) Recovery, boom, contraction, and depression (c) Recovery, boom, contraction, and recession (d) Recovery, contraction, peak, and recession 22. Determining your client's surplus income and how it can best be used is part of which step in the development of a comprehensive strategy? (a) Step 1 - Check that information is complete (b) Step 2 - Confirm the client's current financial position and any financial concerns (c) Step 3 - Establish the client's goals (d) Step 4 - Put in place recommendations to meet the client's desired future financial position

Financial Planning Academy

23. A client has had a long-term asset allocation of 70% growth, 30% defensive. She is looking to increase it to 80% growth, 20% defensive due to the strength of the market. Her adviser suggests she retain her existing allocation. This is an example of: (a) Risk profile allocation (b) Planned allocation (c) Strategic allocation (d) Tactical allocation 24. What strategy do advisers use to ensure that the client's long-term goals are met by directing investments into appropriate asset classes? (a) Risk profiling (b) Tactical allocations (c) Cash flow and budgeting (d) Asset allocation strategy 25. Which of the following best describes a master trust? (a) The investor is the owner of the underlying assets (b) A trustee owns the assets on behalf of the investor (c) The assets are owned by a syndicate (d) The assets are owned directly by the members of the trust 26. Which of the following is a benefit TO THE ADVISER of presenting advice in a written form? (a) It reduces the likelihood of being sued for wrongful advice since, if the advice was appropriate, there is a record of that advice (b) It allows the client to have something to which they can return if they become unsure of the strategy suggested (c) It ensures the adviser is presenting appropriate advice (d) It ensures that the licensee is aware of the advice that is going to be given to the client. If the licensee does not like the advice they can prevent it from being presented

27. In a Statement of Advice, how must fees, commissions and costs be presented?
(a) (b) (c) (d) In both Fees and percentage amounts In fees only In percentage amounts only In either fees or percentage amounts

Financial Planning Academy

28. Your clients appear to be happy with the advice presented to them as they are nodding as each part of the advice is explained. Is the nodding a potential problem and if so, what might you do to overcome it? (a) No, there is no problem with nodding as it shows that the client understands the advice being explained (b) Yes, the client might not understand the advice but doesn't want to show it by asking for clarification. Asking s of the client along the way and encouraging them to ask s will help reveal any lack of understanding (c) Yes, the client might be nodding so as to avoid showing any lack of understanding. Taking the initiative and re-explaining the advice is the best way to handle such a response (d) No, the client is nodding because they understand the advice. Once the advice has been fully presented, give the client time to let it sink in before asking the client to commit to proceeding with the advice 29. The IMPLEMENTATION of a Statement of Advice requires the co-ordination of a number of tasks. These could include: I. An action plan for determining who needs to do what (eg. applications submitted) II. II. Bringing in any specialist professionals as necessary (solicitors, general insurers etc) III. III. Retention of client files for later reference IV. IV. Contacting the client to determine a review date V. (a) I, II and III (b) II and III (c) I, II and IV (d) I, II, III and IV 30. Which of the following is an example of a macro level change that may affect a client's financial plan? (a) A change to marital status (b) An increase in annual living expenses (c) Changes to social security legislation (d) Loss of employment

Financial Planning Academy

Part - 2 1) Money has time value. It derives this value due to existence of several conditions.
Which one of the following is not one of the conditions contributing to the existence of this value? (a) The fees and commission sources of the firm (b) Possibility of increase in tax rates over time. (c) Ability to buy/ rent assets generating revenue (d) Cost of foregoing present consumptions 2) You have term deposits of Rs. 4,00,000 with a bank. In order to meet sudden requirements for liquidity and short-term credit, you are applying for an overdraft facility with the bank. What is the rate of interest you will pay on this facility? (a) The bank will apply a flat rate of interest on the amount of overdraft allowed to actually utilize. (b) The bank will apply a flat rate of interest on the amount of overdraft allowed to you. (c) The bank will apply rate of interest linked to the term deposit rate, on the amount of overdraft utilized. (d) The bank will apply rate of interest linked to the term deposit rate, on the average amount of overdraft remaining unutilized from the OD limit. 3) The Nifty has doubled since the last time you advised your client to reduce his equity exposure. The client is annoyed. What might be the most appropriate action to take immediately? (a) (b) (c) (d) Apologize for wrongly forecasting the market Change his asset allocation by increasing his equity exposure Help the client understand the logic of his asset allocation Rebalance his asset allocation by reducing equity investments

4) A professional indemnity policy protects the insured from risk arising out of _________________. (a) Intentional misconduct (b) Misrepresentation of professional competence (c) Negligence (d) Undisclosed conflict of interest 5) White Knight s a financial services firm that specializes in investment advisory services. In its brochure for Financial Planning services, it may state _____________. (a) It can offer superior investment returns on customer portfolios and talk of the arrangements to offer advice in other areas (b) It has the competence to take care of all Financial advisory requirements of the customer (c) Its competence in investment advisory services and the arrangements to offer advice in other areas

Financial Planning Academy

(d) Its Financial Planning services are the best available in the market in light of its investment advisory capabilities and arrangements to offer advice in other areas 6) Which of the following is a concurrent indicator of the phase of the business cycle? (a) Wholesale price Index (b) Index of Industrial production (c) Labor costs and capacity utilization (d) Order levels in the manufacturing sector 7) What is the main difference between the personal Financial Planning needs of the employed and the self employed ? (a) Attitude to risk/Risk appetite (b) Need to fund childrens education (c) Need to fund retirement (d) The extent of any employer-provided pension benefits 8) Immunization protects bondholders from which of the following risk/s: 1) Interest rate risk 2) Reinvestment rate risk 3) Maturity risk (a) (b) (c) (d) 1 only 2 only 1 & 2 only 1, 2 & 3

9) The modern portfolio theory suggests that the portfolio returns can be optimized by ________________. (a) Investing in diversified equity funds. (b) Investing in treasury bills and equities. (c) Laddering the bond portfolio. (d) Moving closer to the efficient frontier in terms of the risk return equation.

10) The economy is going through a phase of expansion and growth. Industrial production and profitability are high. Your client has a portfolio that is heavily invested in bonds. Which of the following fears of the client is well founded? (a) Higher rates of growth will increase demand for funds and interest rates will firm up, leading to fall in bond prices. (b) Higher rates of growth will require higher imports and expenses. The government deficits will go up. (c) The central bank will try to reduce rates to make funding of business cheaper and reduce costs. That will depress returns on bonds. (d) The currency will become convertible and interest rates will rise as a consequence.

Financial Planning Academy

11) Jubin is a Financial Planner in a large firm. His wife has some large investments in the shares of a few companies. Jubin is required to offer views on almost all of these holdings to clients. Under the Code of Ethics and Rules of Professional Conduct _______ (a) Jubin must disclose the fact to his client(s) so as to make them aware of any potential conflict of interest (b) Jubin has to disclose these holdings only to his employers, if required by the firms internal compliance rules (c) Jubin need not follow any code of ethics and rules of professional conduct. (d) Jubin will not violate the Code and the Rules if he does not disclose his wifes holdings 12) How are financing costs included in NPV and IRR calculations? (a) By including them in the interest payments. (b) By considering the interest rate in the setting of the discount rate (c) As a tax deduction (d) By including them in the earnings 13) Consider a portfolio of two investments viz. A & B. The sum total of volatility of A and B respectively, represented by standard deviation of the two investments, will be equal to the volatility of the portfolio as a whole if _________________. (a) A and B have a correlation of Zero (b) A and B have a correlation of 1 (c) The portfolio is equally divided between A and B (d) The return on the portfolio is equal to the sum of returns of A and B 14) Which of the following is a correct interpretation of the Rules of Conduct pertaining to the Ethic of Confidentiality? (a) A Member must when requested by the client, provide to a person authorized by the client, all original documents prepared or received by the Member in undertaking the advisory task (b) A Member owes to the Members partners or co-owners a responsibility to act in good faith (expectations of confidentiality) only while in business together, not thereafter (c) The Member shall maintain the same standards of confidentiality to employers as to clients (d) Under no circumstance, will any Member divulge any information or knowledge regarding the FPSB India or its members that they may know or be exposed to

15) Mr. Sinhas investment portfolio comprises Rs.2 Lakh in equity, Rs.5 Lakh in debt and
Rs. 1 Lakh in his bank current account. Over one year the returns on equity and debt are 5% and 12%. At the end of the year to maintain his current asset allocation, he needs to _____________. (a) Do nothing. (b) He needs to move Rs, 10000/- from equity and Rs. 60000/- from debt to cash. (c) He needs move Rs.7500/- to equity from debt and Rs. 8750/-to cash from debt (d) He needs to invest Rs. 70000/- in debt and equity.

Financial Planning Academy

16) A 10 year 8.0% bond (Face Value- Rs.1000, interest payable semi-annually) maturing 6 years from today is available at a yield to maturity of 6.0%. It is likely to be priced at _______________. (a) Rs. 1100 (b) Rs. 1149 (c) Rs. 1168 (d) Rs. 1498 17) Spykar a accomplished Financial Planner and is also an expert on derivatives and high yielding bonds. He understands client requirements well and is able to come up with appropriate portfolio restructuring ideas for clients. He believes in quickly moving clients from one investment to another through a dynamic process of research and recommendations. What according to the Rules relating to the Code of Ethics is the most applicable in this case? (a) He does not violate the Rules if he explains to the client the reasons and is able to show that the moves are appropriate to the client (b) He does not violate the Rules since he conducts and has access to research and advises on products relevant to clients based on an understanding of their requirements (c) He does not violate the Rules since he is an acknowledged expert and knows what is best for his clients (d) He violates the Rules as it amounts to active churning of client portfolios 18) Mrs. & Mr. Arora are aged 55 and 58 years respectively. Both expect to work till they turn 65. Their only goal is to fund their retirement. Which of the following is likely to be an appropriate asset allocation strategy for them? (a) 10% sectoral equity, 20% diversified equity, 30% long-term debt, and 40% medium term debt (b) 20% Sectoral equity, 60% diversified equity, 20% long-term debt (c) 30% Sectoral equity, 30% diversified equity, 40% cash/ liquid investments. (d) 80% long-term debt, 20% medium term debt 19) ABC Ltd. is willing to prepay your Cumulative Fixed Deposit with them, without any penalty and with all the accumulated interest (compounded half yearly). You had invested Rs. 4000 with them 3.5 years back. If they are giving you back Rs. 4985, what is the annualized rate of interest you have earned? (a) 6.40% (b) 3.2%. (c) 6.5%. (d) 7.2%. Solution:- 1.) -4000 PV, 4985 FV, 3.5*2 N, Comp I, 2.) Value of I *2 ,to get annualized return.

Financial Planning Academy

10

20) Which of the following is a tort of negligence? (a) Mr. Joy was playing golf. He swings a new golf club on the fairway and the head of the club flies off, and hit another golfer who was standing 20 feet away. (b) Mr. Vishal takes medication that he knows makes him drowsy and then proceeds to drive. He gets into an accident injuring the passengers in another car. (c) Mrs. Jaya locks Ms. Rani in a room to prevent him from leaving the building (d) Mrs. Priti experienced a sudden surge of chest pain while driving, which causes her to lose control of her car and hit another car.

21) Any possible occurrence which may have a negative financial implication, can be plotted on a graph with X axis measuring the frequency (low-high) and Y axis measuring the financial impact (low-high). You can view the classification in four quadrants. Quadrant I - Low frequency, Low Impact Quadrant II - Low frequency, High Impact Quadrant III - High frequency, High Impact Quadrant IV - High frequency, Low Impact It would not be practical to purchase insurance for events falling in _________________. (a) (b) (c) (d) Quadrant I & IV Quadrant I, II & IV Quadrant I, III & IV Quadrant III

22) Karan wants to withdraw Rs. 1200/- at the end of each month for the next 5 years. He expects to earn 10% interest compounded monthly on his investments. What lump sum should he deposit now? (a) Rs. - 56949 (b) Rs. - 58630 (c) Rs. - 56478 (d) Rs. - 59119 HINT :- 1200 PMT, 5*12 N, 10%/12 I, Comp PV

Financial Planning Academy

11

Part - 3
Q1. The body responsible for the formulation and implementation of monetary policy in India is ___________________________. A. Reserve Bank of India B. SEBI C. Bank of India D. Institute of Economic Research. Q2. Which of the following statements does not reflect the meaning of financial planning? A. Determine your specific financial goals and objectives. B. Design strategies to attain your financial objectives. C. Analyse your present financial position. D. Invest in securities that provide the highest return. Q3. Calculate the yield to maturity of a bond with the following details: Face Value : 1,000.00 Market Price : 950.00 Coupon Rate (paid annually) : 5.0% Remaining Te to Maturity : 6 years (a) 5.16% (b) 4.29% (c) 5.00% (d) 6.02% HINT :FV = 1,000, PV = - 950, Pmt = 50, N = 6, Compute I% = 6.02% Q4. All the following assets are generally considered to be protected from a decline in purchasing power due to increase in inflation except: A. The husbands employment income. B. The wifes rental income. C. The wifes dividend income. D. The husbands pension income.

Q5. All the following are characteristics of a typical risk averse except: I. Overestimation of risks. II. Preference for certainty. III. Over-optimism. A. I, II B. II, III C. I D. III

Financial Planning Academy

12

Q6. Ali owns a life insurance policy for 100,000 on his life and named his sister as beneficiary. If Ali dies, how much of that 100,000 is included in his estate? A. Depends on the amount agreed by the beneficiary. B. Nothing, Alis sister is the beneficiary of that sum insured. C. Half of it. D. All of it. Q7. Alok purchased an insurance policy on his life that requires ten equal annual premiums. The policy provides a 100,000 death benefit if Alok dies within ten years of purchasing the policy, as long as he continues to pay the annual renewal premiums. All forms of coverage cease after the 10-year period. The type of insurance purchased by Alok is: A. Disability income insurance. B. Term insurance. C. Whole of life insurance. D. Participating endowment insurance. Q8. A corporation proposes to issue a 5-year bond with a coupon rate of 8.0%. The prevailing yield to maturity of bonds with similar risk and term is 10.0%. The bond will sell `____________ to its face value. A. At a premium. B. At a discount. C. At par. D. At some indeterminate value. ( FV = 1,000 N = 5 Pmt = 80 (8% of 1000) I = 10% Compute PV = 924.18, a discount to face value ) Q9. Identify which amongst the following represents a personal risk: A. The economic loss to the owners of a factory destroyed by a fire. B. The economic loss to a husband when his wife is taken ill. C. The economic loss to the owners of a firm brought about by a products liability suit. D. The economic loss to a physician brought about by a professional indemnity suit. Q10. All of the following changes in planning assumptions will increase the amount a person will need in order to achieve his retirement goals except: A. An increase in the rate of inflation. B. A decrease in the rate of investment earnings. C. A decrease in the planned retirement age. D. A decrease in the mortality age. Q11. Which of the following is not a characteristic/feature of a Will? A. Serves as a declaration of intent only. B. Must be in writing. C. Only takes effect upon the death of testator. D. Revocable by divorce.

Financial Planning Academy

13

Q12. Which of the following may not be considered as essential characteristics of a sound financial plan? A. It must be tailored towards achieving clients objectives. B. It must provide measures to guarantee attainment of goal. C. It must be within clients resources to implement the plan. D. It must clearly identify the roles of the implementers. Q13. Which of the following is/(are) considered quantitative information that needs to be gathered by a financial planner from a client: I. Assets and liabilities. II. Insurance policies. III. Investment portfolio. IV. Family relationships. A. I, II B. I, II, III C. I, II, IV D. All of the above.

Q14. Why is it necessary for a financial planner to establish a well-defined client-planner relationship? I. To prove that the planner is an honest person. II. To instill a sense of trust between them. III. To develop a sense of confidence in the planners ability. IV. To facilitate discussion on personal issues. A. I, II B. II, III C. II, III, IV D. All of the above.

Q15. The following are the six basic steps in the financial planning process: I. Gathering information and establishing goals. II. Establishing and defining client-planner relationship. III. Create a financial plan. IV. Analyze information. V. Implement the plan. VI. Monitor the plan. What is the proper sequence of the process? A. I, II, III, IV, V, VI B. II, I, III, IV, V, VI C. II, I, IV, III, V, VI D. II, I, IV, III, VI, V

Financial Planning Academy

14

Q16. Why is it important for a financial planner to practice good professional ethics? I. To maintain continued trust on the profession. II. To avoid being the case of one rotten apple spoils the entire basket. III. To avoid intervention from regulators. A. I B. I, II C. I, III D. All of the above. Q17. The rule that says a CFP designee shall satisfy all minimum continuing education requirements established for CFP designees by FPAM is related to which principle in the Code of Ethics? A. Principle of integrity. B. Principle of objectivity. C. Principle of competence. D. Principle of fairness. Q18. The following are some of the principles a professional financial planner should observe in performing his fiduciary role, except:: A. Duty to disclose information on services offered, etc. B. Duty to diagnose clients financial position and make recommendation. C. Duty to keep abreast with current development in rules and regulations. D. Duty to implement the agreed plan. Q19. The confidentiality principle requires a financial planner to do the following, except: A. Not to disclose a clients information to a third party without the clients consent. B. Disclose clients information only in response to a proper legal process. C. Use clients information only for the purpose of preparing the financial plan. D. Treat clients in the same manner the planner wants to be treated.

Financial Planning Academy

15

Part - 4
1. Which of the following is true? A. A professional financial planner provides only comprehensive financial advice to clients encompassing, estate planning, insurance risk management, income and expenditure (cash flow), retirement benefits, investment planning B. A professional financial planner may provide limited advice, if he discloses the fact at the outset to the client. (a) (b) (c) (d) Both A and B are false A is true but B is false A is false but B is true A is true provided, the financial planner is a CFPCM certificant

2. A financial planner who receives commission from companies on sale of investment/insurance products to a client is being unprofessional. (a) The above statement is true (b) The above statement is false (c) The above statement is false, provided the financial planner discloses the fact to the client at the beginning of the relationship. (d) The above statement is true, provided he also charges service fees from the client. 3. A financial planner in Australia may call his/her services ' independent' provided (a) He/She avoids commissions/trailing commissions, soft dollar arrangements and other benefits from product providers which may tend to create a product bias (b) Operates free from any direct or indirect restrictions relating to the securities recommended (c) Operates without any conflict of interest by ownership links to product providers (d) All of the above 4. The most common client in the early years of financial planning internationally was in the age bracket: (a) 25-35 (b) 55-65 (c) 45-55 (d) Above 70

5. 'Empty nesters' usually fall in the age bracket


(a) (b) (c) (d) 55-65 35-45 45-55 They could fall in any age bracket

Financial Planning Academy

16

6. Financial planners are trained to write wills and/or sell insurance (a) The above statement may be true (b) The above statement is false (c) The above statement is true in all cases (d) The above statement is true if the financial planner is a CFPCM certificant 7. By comprehensive financial planning we mean (a) Financial planning by a team of persons (b) Financial planning by a CFP CM certificant (c) Planning which encompasses all areas of a client's life (d) Financial advice on cashflows & budgeting, insurances, retirement planning, investments, tax planning and estate planning. 8. A statement of advice is not needed in providing limited financial planning advice. (a) There is no such thing known as a statement of advice (b) The above statement is true (c) The above statement is false (d) There is no such thing as limited financial planning advice, by definition financial planning is comprehensive. 9. There are usually _______ meeting/s before a financial plan can be implemented. (a) One (b) Two (c) Three (d) Four 10. A professional financial planner is one who (a) Takes pride in his/her work (b) Is committed to quality (c) Is dedicated to the interest of the client (d) All of the above 11. In India, formalized/written complaints handling procedures for financial planning businesses are a must. This is a requirement of (a) The law (b) The FPSB (c) It is not a requirement (d) It is expected to be introduced shortly in the law. 12. For a successful financial planner (a) Technical skills are more important than people skills (b) People skills are more important than technical skills (c) Both are equally important (d) Neither skill is an essential pre-requisite

Financial Planning Academy

17

13. The most valuable asset of a financial planner/planning firm is: (a) The number of clients (b) The funds under management (c) Customer satisfaction (d) Trust 14. It is sound business practice to fill up the client questionnaire at the first meeting with the client (a) True (b) False (c) Give the questionnaire to the client to fill at home at his/her leisure. (d) Could be filled up at any meeting 15. At the first meeting with the client (a) Get straight to the point (b) Break the ice with small talk and a cup of tea (c) Explain your role and the scope of the engagement (d) b and c 16. Under the rules of professional conduct of the FPSB, a planner may charge (a) Any amount of fees (b) Only service fees (c) Only investment placement fee (d) Any fees provided it is fair and reasonable 17. According to the FPSB rules of professional conduct, disclosure regarding compensation needs to be made only at the time of establishing the relationship with a new client. (a) True (b) False (c) Sources of compensation need not be disclosed (d) Need to be disclosed whenever there is a change in status. 18. Telling a client about research capabilities or the use of computers in your financial planning firm amount to (a) Unprofessionalism (b) Advertising (c) Smart thinking (d) Waste of time 19. According to Ross Levin- CFP, there are _______ types of clients (a) Three (b) Four (c) Five (d) Six

Financial Planning Academy

18

20. Sometimes clients expect planners to solve every financial problem they have like overspending, losses in the share markets and want to earn 20-30% p.a. etc. (a) True (b) False (c) True only according to Mary Rowland (d) False according to Mary Rowland 21. If a planner does not receive sufficient and relevant information from a client he/she should: (a) Terminate the relationship (b) Give restricted (limited) advice (c) Go ahead but give a disclaimer disclaiming all responsibility (d) Either a or b 22.Gathering comprehensive information on the client is useful because: (a) It is in compliance with the AFP rules of professional conduct (b) It improves your legal position in case of a legal suit (c) It makes good business sense (d) All of the above 23. 1) Investment experience, 2) Investment time horizon 3) Concern liquidity/flexibility are all examples of qualitative information about a client. (a) True (b) False (c) Only 1) and 3) are true examples (d) None of them are examples 24. Risk profiling is (a) A method to determine a client's attitude to risk (b) A method to determine the risk underlying a client's portfolio (c) Measurement of the risk of losing a client (d) Measurement of the risk of erosion of a client's networth 25. It is best to start a data gathering client interview with (a) Qualitative details (b) Client's finances (c) Planner's fees details (d) Quantitative information 26. A planner should approach other professionals associated with a client through (a) The client by taking him/her along (b) Authority letters from the client (c) Directly (d) Need not approach them as the client can gather all the information. for

Financial Planning Academy

19

27. Financial planning needs can be categorized into two broad areas: (a) Investment needs & insurance needs (b) Insurance needs and retirement needs (c) Needs for predictable events and needs to provide for unpredictable events (d) Investment needs and retirement needs 28. Which of the following is true ? (a) Needs take precedence over wants (b) Needs are the same as wants (c) Financial planning addresses both needs and wants (d) a and c 29. A financial planner should (a) Confine himself strictly to the client's brief (b) Point out all flaws in a client's financial position that he may notice (c) Not agree to providing limited or restricted advice (d) Always provide comprehensive financial planning advice 30. The client questionnaire records quantitative data and _______ record/s qualitative data. (a) The planner's mind (b) File notes (c) The same client questionnaire (d) A separate questionnaire 31. As part of their job, financial planners have to predict future economic indicators which impact clients. (a) The above statement is true (b) The statement is false (c) It is true in cases of economic turmoil (d) The statement is true for CFP CM certificants 32. Leveraged investments are (a) Investments bought with debts raised (b) Options, futures etc. (c) Derivatives (d) b and c 33. There are two types of product research: (a) Generic and new product research (b) Old and new product research (c) Investment and insurance product research (d) There are nil types of product research

Financial Planning Academy

20

34. Planners may keep abreast of new products and developments through (a) Perusal of the press (b) Industry functions and seminars (c) Fund manager briefings (d) All of the above 35. Micro-economics refers to the study of economics at (a) National level (b) Level of the firm (c) Personal level (d) b and c 36. The Indian economy lies in-between a capitalist economy and a socialist economy. (a) True (b) False (c) Partially true (d) True but it is now moving towards a socialist economy. 37. ________ is India's largest trading partner (a) USA (b) Russia (c) UK (d) Germany 38. The fastest growing sector in the Indian economy is the (a) Primary sector (b) Secondary sector (c) Tertiary sector (d) None of these 39. Construction is classified as a (a) Service industry (b) Manufacturing industry (c) Agricultural activity (d) None of the above 40. If the economic symbols have their usual meanings, GDP = Rs. 8000 Bn., C= Rs. 2000 Bn., I = Rs. 1000 Bn., G = Rs. 4000 Bn. and X = Rs. 2000 Bn. then M = (a) Rs. 2000 Bn. (b) Rs. 3000 Bn. (c) Rs. 1000 Bn. (d) Rs. 500 Bn. {Hint: GDP= C+I+G+(X-M)}

Financial Planning Academy

21

41. For living standards in India to rise, (a) GDP must grow (b) Population and GDP, both must grow (c) GDP must grow at a higher rate than population (d) GDP must grow while population should fall 42. High level of employment, faltering business, slackening rate of investment activity, higher costs for business firms are indicators of which stage of the business cycle (a) Contraction (b) Recession (c) Recovery (d) Boom 43. Inflation is important to financial planners because (a) It serves as a benchmark for capital growth in an investment portfolio (b) It serves as a benchmark for income growth in an investment portfolio (c) It indicates that the economy is growing (d) It indicates that money should not be kept in banks 44. Economic indicators which point to the current state of the economy are called (a) Leading indicators (b) Lagging indicators (c) Coincident indicators (d) None of the above 45. M3 growth refers to (a) Growth in money supply (b) Growth in GDP (c) Growth in inflation (d) Growth in demand for money 46. Government policy which regulates interest rates in the economy to control money supply and hence inflation is called (a) Fiscal policy (b) Monetary policy (c) Interest rate policy (d) None of the above 47. The advantage with monetary policy is that (a) It can be implemented reasonably quickly (b) It is more effective than fiscal policy (c) Interest rates are more important than taxes (d) It influences money supply, a key variable

Financial Planning Academy

22

48. The bank rate, CRR and SLR are key determinants of the long-term interest rates in the economy. (a) The above statement is true (b) The above statement is partly true (c) CRR and SLR do not affect interest rates (d) The above statement is false 49. If the long term interest rate sought is 9%, the real rate of interest is 6%, then inflation is likely (a) 0.15 (b) 0.03 (c) 0.04 (d) 0.06 50. Fiscal policy affects the economy in that: (a) Higher government spending can stimulate the economy (b) Increases in personal taxes can dampen sentiment and hence consumer demand (c) Fiscal deficit funded through borrowings by issuing government securities may raise interest rates (d) All of the above 51. Over the past few years, the rupee has depreciated against the dollar which has benefited (a) Exporters (b) Importers (c) Both (d) Neither 52. A higher exchange rate can (a) Increase GDP (b) Have no effect on GDP (c) Decrease GDP (d) Improve India's export competitiveness 53. The body formed on abolition of the CCI was (a) RBI (b) SBI (c) IRDA (d) SEBI 54. The apex bodies for the co-operative banks in India is/are (a) Government (b) RBI (c) NABARD (d) All of the above

Financial Planning Academy

23

55. The leading credit rating agency in India is the (a) ICRA Ltd. (b) CARE (c) Fitch Ratings, India (d) CRISIL 56. A ____ % cap has been kept on foreign shareholding in an insurance company (a) 26 (b) 51 (c) 75 (d) 40 57. The first step in the strategy development process is to: (a) Check that you have all the information (b) Secure the client's current financial position (c) Establish the client's goals and financial concerns (d) None of the above 58. When you establish the client's goals, you (a) Need to keep them as few as possible (b) Should encourage the client to take on debt to meet all their goals (c) Need to analyse their current position to ensure fulfillment (d) Encourage the client to fulfill their dreams 59. To meet short term objectives, it is best to invest in (a) Equity shares (b) Bonds (c) Govt. securities (d) Cash and fixed interest type investments 60. To meet long term objectives, it is most essential to keep in mind (a) Income returns from the portfolio (b) Capital growth in the portfolio (c) Liquidity of the portfolio (d) Volatility of the portfolio 61. Asset allocation strategy is guided by (a) Portfolio diversification (b) Objectives of the client (c) Client's risk profile (d) All of the above

Financial Planning Academy

24

62. In taking investment decisions, you should directly select specific investment products for recommendations to the client. (a) The above statement is partly true (b) The above statement may be true in some circumstances (c) The above statement is true (d) The above statement is false 63. According to financial consultants, Godfrey Pembroke, a balanced portfolio will not show any negative total returns. (a) False (b) True (c) True if at least 50% is in shares (d) True if at least 50% is in debt 64. While selecting investment products, one must (a) Quantify the risk, returns and time frame (b) Provide for an emergency fund (c) Diversify (d) All of the above 65. While developing a financial plan, (a) Financial planning software should not be used (b) Financial planning software may be used solely to check your calculatons (c) Financial planning software may be used as a support, specially for financial mathematics (d) Financial planning software can write the whole financial plan 66. A diversified portfolio may still not reduce risk significantly (a) False (b) True (c) Diversification has nothing to do with risk reduction (d) Risk in a portfolio cannot be reduced significantly 67. A financial plan should always (a) Be verbal or written (b) Be verbal (c) Be comprehensive (d) Be written 68. Some of the important advantage/s of a written financial plan is/are (a) That the client can suggest changes to the plan (b) That the client can raise doubts and questions on the plan (c) That the client cannot proceed legally against the planner (d) All of the above

Financial Planning Academy

25

69. Which of the following is false (a) Significant advantages accrue to a financial planner through a written financial plan (b) Significant advantages accrue to the financial planning firm through a written financial plan (c) Significant advantages accrue to the client through a written financial plan (d) All of the above 70. The following is not an essential component of a financial plan: (a) Executive summary (b) Financial Planning strategy (c) Letter of engagement (d) Summary of services provided 71. The engagement letter should be taken at the time of the first presentation of the plan to the client. (a) True (b) False (c) The letter of engagement need not be taken from the client (d) Not true because the plan may be sent to the client by post 72. 'The financial planner/financial planning firm is/are not responsible for consequences arising out of the action taken on the basis of the financial plan' is an example of a poor disclaimer (a) True (b) False (c) False, only if the financial planner is a CFP CM certificant (d) True, only if the financial planner is a CFP CM certificant 73. There is no need to clutter the financial plan with your calculations/analysis (a) True (b) False (c) False, only if it is a new client (d) True, you should keep the calculations only for possible use in case of litigation. 74. According to financial planning best practice standards, plans generated on financial planning software are: (a) Acceptable (b) Unacceptable (c) Acceptable, if the planner is not a CFPTM certificant (d) Acceptable, if the software also generates commentary and produces word-processed documents

Financial Planning Academy

26

75. Before presenting the plan to the client, the planner needs to (a) Prepare the plan document (b) Prepare for the presentation meeting (c) a and b (d) None of the above 76. Presentation of the plan to the client includes: (a) Explanation of the cost structures associated with the financial plan and its implementation (b) Client declaration (c) Authority to proceed (d) All of the above 77. Usually a time lag of ______ week/s is recommended between the tabling of the plan and the next meeting with the client. (a) One (b) Two (c) Three (d) Four 78. Any changes in recommendations suggested by the client (a) Should be vehemently opposed by the planner (b) Should be rejected outright (c) Should be incorporated in the plan (d) If incorporated, should be confirmed in writing including reasons thereof 79. Once a client is ready for implementation of the plan, there is need to prepare (a) A letter of engagement (b) An Authority to proceed (c) An Action plan (d) Authorization letters 80. As a CFP CM certificant, best practice standards require that you (a) Keep only computer files (b) Keep only paper files (c) Keep both computer and paper files (d) Files need to be maintained at the client's end. 81. At the macro level, (a) Share markets may rise or fall (b) Wages and/or business income may rise or fall (c) The client may lose his/her employment (d) None of the above

Financial Planning Academy

27

82. It makes enormous commercial sense to provide an ongoing financial planning service to clients (a) False (b) True (c) It is easier to attract new clients (d) The service is provided because it is mandatory under the AFP rules of professional conduct 83. A strategic review of a client' s situation is required in case of (a) Macro level changes (b) Micro level changes (c) Neither (d) Both 84. The most appropriate criterion for deciding on the frequency of portfolio reviews of a client is: (a) Client profile (b) Funds under management (c) Fees received from client (d) Planner's discretion 85. The key difference between marketing share broking services and marketing comprehensive financial planning services is that: (a) Share broking is a purely transaction driven business (b) Shares are easier to sell (c) Comprehensive financial planning is easier to sell (d) There is no difference 86. A CFPCM certificant in India need not disclose the reasons for moving from one investment to another for a client (a) The above statement is true (b) He/she has to disclose it under law (c) He/she has to disclose it under FPSB rules of professional conduct (d) He/she just has to inform the client of the change 87. A letter of engagement is: (a) A legal document (b) A requirement of the FPSB rules of professional conduct (c) Essential for client-planner relationships (d) Like a memorandum of understanding

Financial Planning Academy

28

88. A review exercise for a client consists of: (a) A strategic review (b) A portfolio review (c) Neither (d) Both 89. To decide on the frequency of reviews for a client, the planner has to consider: (a) The level of funds under management (b) The type of investment portfolio (c) The 'pace of change' in the client's circumstances (d) All of the above 90. Tool/s that can aid a financial planner in conducting reviews/monitoring of client portfolios are : (a) Portfolio management systems (b) Investment research (c) a and b (d) An efficient secretary 91. A 'charge' over an asset is (a) An extra payment to be made to acquire the asset (b) Rental for the asset (c) Same as leasing the asset (d) The right to dispose off the asset by the lender in the event of non-repayment of loan 92. The difference between a mortgage and an overdraft is that: (a) An overdraft is available for a year (b) An overdraft may be unsecured (c) A mortgage usually has a lower rate of interest (d) All of the above 93. For a self-employed professional, income proof is usually a/an (a) Statement of accounts (b) Remuneration slip (c) Income-tax return (d) Any of the above 94. The drawback/s with credit cards is/are that (a) They are cumbersome to use (b) They charge a high rate of interest (c) With rollover credit, there is danger of falling into a debt trap (d) b and c

Financial Planning Academy

29

95. An income and expense statement for a client is needed to determine (a) Asset base (b) Liabilities & Debts (c) Taxation (d) Cashflow 96. While income tax is an example of direct taxes, indirect taxes are (a) Excise duty (b) Customs duty (c) Local sales tax (d) All of the above 97. If 2006-07 is the previous year, the assessment year would be (a) 2007-08 (b) 2001-02 (c) 2000-01 (d) None of the above 98. The surcharge on income tax is now: (a) 0.02 (b) 0.03 (c) 0.04 (d) 0.10 99. Under Section 80CCC(1), deduction is available for (a) Interest income (b) Dividend income (c) A notified pension plan (d) Income from house property 100. The difference between a deduction and a rebate is that (a) A deduction is a reduction from assessable income while a rebate is a reduction from tax payable (b) Both are same (c) A deduction relates to salary income while a rebate relates to income from business or profession (d) A deduction does not affect tax paid while a rebate does. 101. OASIS stands for (a) Old Age Social & Income Security (b) Old age Social & Investment Scheme (c) Old Age Security Income Scheme (d) None of the above

Financial Planning Academy

30

102. Internationally, there is a trend towards (a) Dismantling defined contribution funds (b) Dismantling defined benefit funds (c) Neither (d) Both a and b 103. Under a statutory provident fund, all incomes including interest are exempt from tax. (a) The above statement is true (b) These tax breaks are available to a recognized provident fund (c) These tax breaks are available to an unrecognized provident fund (d) None of the above 104. No contribution from employees or employers is required in the case of: (a) Employees' provident fund scheme (b) Employees' pension scheme (c) Employees' deposit linked insurance scheme (d) None of the above 105. A public provident fund account can be closed after: (a) 4 years (b) 5 years (c) 10 years (d) 15 years 106. For a defined contribution superannuation fund, contribution is tax exempt upto (a) 5% of salary (b) 8% of salary (c) 10% of salary (d) 15% of salary 107. For the purpose of calculating the tax benefit of gratuity, a month is assumed to contain (a) 22 working days (b) 28 working days (c) 26 working days (d) 30 working days 108. Standard deduction is available for (Not Applicable from AY 2006 -07) (a) Salary (b) Pension (c) a and b (d) Neither

Financial Planning Academy

31

109. Money market comprises (a) Short term investments (b) Long term investments (c) Cash (money) only (d) None of the above 110. Fixed interest sector consists of debt instruments with a maturity of (a) More than one year (b) 1-3 years (c) More than 3 years (d) None of the above 111. If you invest in the fixed interest sector (a) You will always earn a fixed rate of return (b) You will earn a fixed rate of interest (c) You will have enormous capital growth potential (d) None of the above 112. If interest rates are rising, prices of fixed interest securities will (a) Rise (b) Not be affected (c) Fall (d) None of the above 113. A sole proprietorship is governed by the (a) Negotiable Instruments Act (b) Indian Trusts Act (c) Contract Act (d) None of the above 114. The minimum number of members in a public limited company is (a) 10 (b) 20 (c) No limit (d) 7 115.A co-operative type of organization is useful for (a) Large businesses (b) Small businesses (c) Small and medium sized businesses (d) None of the above

Financial Planning Academy

32

116.If the number of members in a Private Limited Company goes above _____, a public limited company may be required (a) 100 (b) 1000 (c) 50 (d) 150 117. Initial public offerings (IPO) for shares are part of (a) Secondary market for shares (b) Primary market for shares (c) Neither , have a separate market (d) Derivatives market 118. Problem/s that plague Indian capital markets are (a) Illiquidity (b) Shallowness (c) Inadequate disclosure standards (d) All of the above 119. Mutual funds are advantageous because (a) Of diversification of risk (b) Access to otherwise inaccessible markets (c) Easy liquidity (d) All of the above 120. A mutual fund may also be disadvantageous because (a) Risk is higher in a mutual fund (b) You do not have any ownership rights in the fund (c) Management fees may be high (d) It may not declare any dividend 121. Assets underlying derivatives are: (a) Shares (b) Bonds (c) Other tradeable securities (d) Any of the above 122. The standard deviation of returns of a financial instrument denotes (a) Risk (b) Return (c) Price (d) Its nature

Financial Planning Academy

33

123. The higher the risk, (a) The higher the return (b) The lower the return (c) The return is independent (d) The less volatile the return 124. Determining a client's attitude to risk is known as (a) Data gathering (b) Risk determination (c) Risk profiling (d) Qualitative data gathering 125. The best methods of determining a client's attitude to risk are (a) Quantitative (b) Qualitative (c) A mixture of both (d) None of the above 126. A client's attitude towards risk is likely to: (a) Change over time (b) Remain stable over time (c) Remain stable, unless there is a major change in the client's situation (d) None of the above 127. Speculative risk refers to risk (a) Where there may be a loss or no loss (b) Where there is a loss or a gain (c) Where there may be a gain or no gain (d) All of the above 128. Disadvantage/s of investing in cash or fixed interest type of investments are that: (a) They may be subject to fluctuations in total return (b) They show little protection against inflation (c) a and b (d) None of the above 129. A property trust is (a) A sound property investment (b) A scheme of bequeathing a property (c) Trust in property investments (d) A pooling of funds in property investments for mutual benefit

Financial Planning Academy

34

130. As a part of risk management, you may decide to retain some part of risk. (a) True, if you have surplus funds (b) True, if you are short of funds (c) True (d) False 131. Some personal risk/s that people face is/are (a) Risk of early death (b) Risk of living too long (c) Risk of injury (d) All of the above 132. From middle age, a client may need (a) Income protection insurance (b) Life insurance (c) Critical illness insurance (d) Total and permanent disability insurance 133.Trauma insurance covers (a) Total and permanent disability (b) Terminal illness (c) Life threatening medical condition (d) None of the above 134.The following type of policy does not have a savings/investment component (a) Whole of life (b) Endowment (c) Convertible whole life (d) Term insurance 135.A life insurance policy may be 'encashed' earlier through a (a) Surrender value (b) Policy loan (c) A and b (d) None of the above 136.As opposed to life insurance, general insurance relates to (a) The individual (b) Tangible property (c) Property, whether tangible or intangible (d) Intangible property

Financial Planning Academy

35

137.A General Insurance policy generally (a) Places the insured in the same position as before the loss (b) Places the insured in a better position than before the loss (c) Places the insured in a position which may be worse off than before the loss (d) May be any of the above 138.A major difference between life and general insurance is that (a) Unlike general insurance, a life insurance contract has to be renewed each year (b) Unlike life insurance, a general insurance contract has to be renewed each year (c) Life insurance does not cover risk while general insurance does (d) General Insurance does not cover risk while life insurance does 139.A major difference between life and general insurance is that (a) Unlike general insurance, a life insurance contract may contain an investment component (b) Unlike Life insurance, a general insurance contract may contain an investment component (c) Neither (d) A general insurance contract is for large amounts while a life insurance is for small amounts 140.In India, health insurance is the subject matter of (a) Life insurance (b) General insurance (c) Neither, it is a separate head (d) Mediclaim is covered under life insurance while other schemes come under general insurance 141.If an individual dies without drawing up a will, he is said to have died (a) A widower (b) Without a beneficiary (c) Intestate (d) Without an executor 142.A will may require a change in the event of (a) Death of beneficiary (b) Marriage or divorce (c) Significant change in financial position (d) All of the above 143.A financial planner may prepare a will for a client provided he is a (a) CFPCM certificant (b) A lawyer (c) A chartered accountant (d) None of the above

Financial Planning Academy

36

144.There are mainly two types of powers of attorney, one is a general power of attorney, the other is a (a) Specific power of attorney (b) Residual power of attorney (c) Special power of attorney (d) None of the above 145.A person who carries out the instructions in a will on behalf of the deceased is called (a) A Beneficiary (b) A Testator (c) An Executor (d) A Lawyer 146.ALC in the context of cashflow planning and budgeting refers to (a) Average living costs (b) Annual living cashflow (c) Annual living costs (d) None of the above 147.A financial planner should (a) Always suggest his own cashflow management system to the client (b) Not advise on cashflow planning (c) Improve upon an existing system if it is adequately effective otherwise (d) None of the above 148.For a retired client, a main account (e.g. a 2-in-1 account) is needed because (a) It pays a higher rate of interest (b) It evens out uneven cashflows (c) Both a and b (d) None of the above 149.If the inflation rate is 3% and the tax rate is 40%, the required rate of return to maintain the value of an investment is (a) 0.04 (b) 0.05 (c) 0.06 (d) 0.07 150.There is a need for a thorough evaluation of a company before investing in its shares. This is because (a) Shares represent part-ownership of a business (b) Shares are risky investments (c) Shares involve heavy cash outlay as compared to other investment avenues (d) a and b

Financial Planning Academy

37

151.A company's relative capital structure can be known from its (a) Debt-Equity ratio (b) P/E ratio (c) Liquidity ratios (d) None of the above 152.A popular measure of the valuation of a company's share on the stock markets is (a) Dividend yield (b) Gearing (c) Earnings yield (d) P/E ratio 153.The most important advantage of equity shares is that (a) They provide long term growth over inflation (b) They have low risk (c) They provide risk diversification (d) All of the above 154.An investment assets portfolio should have (a) Sector risk diversification (b) Legislation risk diversification (c) Company or product risk diversification (d) All of the above 155.A retiree should not invest in (a) Equity shares/mutual funds (b) Fixed deposits of banks (c) Govt. guaranteed bonds (d) None of the above 156.An effective equity share investment strategy is to time the market (a) The above strategy is difficult to implement in practice (b) It is more effective to invest for the long term (c) It is more effective to diversify your portfolio (d) All of the above 157.In the interest of your clients, for tax advice (a) Refer them to a tax consultant (b) Refer them to a CFP CM certificant (c) Seek expert advice yourself before recommending strategies (d) None of the above

Financial Planning Academy

38

158.The Indian taxation system is: (a) Progressive (b) Regressive (c) Assertive (d) None of the above 159.Capital assets exempted from capital gains tax include (a) Jewellery (b) Rural agricultural land (c) Shares (d) None of the above 160.An equity share held for 12months is (a) A long term capital asset (b) A short term capital asset (c) Not a capital asset (d) None of the above 161.A distinction is made between long term and short term capital gain because (a) Long term capital gains are taxed at a lower rate of tax (b) Deductions under section 80CCC to 80U are not available against long term capital gains (c) a and b (d) None of the above 162.If you avail indexation benefit on long-term capital gains in respect of listed equity shares, where STT is paid then, the rate of tax is (a) 10 (b) 30 (c) NIL (d) 20 163. Long term capital gains tax can be saved if the capital gains is invested in Bonds issued by (a) National Highways Authority of India (b) Rural Electrification Board (c) Small Industries Development Bank of India (d) Either a or b 164. Long term capital gains can be saved by investing the gain an IPO (a) The above statement is true (b) The statement is false (c) Even short term capital gains can be saved the same way (d) None of the above

Financial Planning Academy

39

165.Income may be split between spouses (a) By diverting salary income from one spouse to another (b) By splitting investment income (c) By transferring assets as well as income to the other spouse (d) None of the above 166.In an effective gifting strategy for tax planning, the gift should be (a) Reciprocated (b) Irrevocable (c) Complementary (d) None of the above 167.A partnership is an effective tax planning strategy if (a) All the partners provide services (b) The partners are spouses (c) The partners are family members (d) One of the partners is a sleeping partner 168.Growth based investments, effectively (a) Lower taxes over the long term (b) Defer taxes (c) Avoid taxes (d) a and b 169.Interest on borrowings for investment is (a) Not allowed as a tax deductible expense (b) Allowed as expense for tax purposes (c) Used as a tax planning strategy (d) b and c 170.One of the major common risks of tax planning is that (a) Legislation changes affecting taxes are frequent (b) You may end up paying higher tax (c) You may be prosecuted for tax evasion (d) All of the above 171.An effective retirement planning strategy should commence when (a) The person is born (b) When he is an adult (c) When he joins the workforce (d) When he gets married

Financial Planning Academy

40

172.For an effective retirement planning strategy, one should consider (a) Age of retirement (b) Health of retiree (c) Desired lifestyle in retirement (d) All of the above 173.One of the risks inherent in a retirement plan include: (a) Inadequate medical insurance (b) Early death (c) Lack of a will (d) All of the above 174.Insurance covers (a) All risks (b) Pure risks (c) Impure risks (d) None of the above 175.The following risks may not be covered by insurance: (a) Being sued for slander (b) Loss of income through injury/illness (c) Disruption of business through malicious damage (d) None of the above 176.Life insurance is advantageous because (a) It provides liquidity through loans (b) A beneficiary gets the life insurance benefit immediately on death of life insured (c) Proceeds from surrender or maturity of a life insurance policy are exempt from capital gains tax. (d) All of the above 177.If a person is 45 years of age and his gross annual income is Rs. 2,00,000, his life should approximately be insured for (a) Rs. 30,00,000 (b) Rs. 4,00,000 (c) Rs. 15,00,00 (d) Rs. 20,00.00 ( Assumed Approx. 10 times of annual income.) 178.Apart from rules of thumb, the other approach to estimate insurance needs is (a) Estimate approach (b) Needs approach (c) Necessity approach (d) None of the above

Financial Planning Academy

41

179.The following are usually attached to life insurance policies as riders: (a) Accident benefit (b) Critical illness (c) Income protection (d) Either a or b 180.In case a client owns a business, the financial planner needs (a) To seek expert advice (b) To consult a CFP CM certificant (c) To be extra careful in giving recommendations (d) All of the above 181.Insurance to cover doctors, lawyers, CAs etc. against professional negligence is called (a) Professional liability insurance (b) Professional indemnity insurance (c) Professional negligence insurance (d) None of the above 182.The householder's insurance policy covers property risk of (a) Fire (b) Theft (c) Damage in riots (d) All of the above 183.It is important to insure (a) All risks (b) Some risks, some to be retained (c) Risks depending on the individual (d) None of the above 184.Estate planning objectives would include: (a) Providing for dependents (b) Minimization of taxation (c) Satisfaction of philanthropic objectives (d) All of the above 185.Estate planning strategies need to be (a) Rigid (b) Changed frequently (c) Flexible (d) Dynamic

Financial Planning Academy

42

186.If a person dies intestate, the assets are distributed (a) According to respective succession acts (b) By the courts (c) By the deceaseds lawyer (d) To charity 187.A will may need changes in case of (a) Marriage (b) Divorce (c) Re-marriage (d) All of the above 188.For estate planning, a CFP CM certificant should (a) Refer the preparation of a will to a lawyer (b) Know the contents of a will (c) Be aware of estate planning issues (d) All of the above 189.Some of the tax benefits available in estate planning are (a) Transfer of capital assets under a will are exempt from capital gains tax (b) Trust created under a will for a dependent relative is exempt from tax (c) Both a and b (d) No tax benefits are available for estate planning 190.The following is/are an important component of estate planning (a) Business succession planning (b) Life insurance policy (c) A and b (d) None of the above 191.The sponsor of a mutual fund is its (a) Manager (b) Trustee (c) Promoter (d) None of the above 192.The AMC of a mutual fund reports to (a) The sponsor (b) AMFI (c) SEBI (d) The trustee company

Financial Planning Academy

43

193.The management fees of an AMC (a) Is flexible (b) Is fixed (c) Changes with funds under management (d) Is decided by SEBI 194.The NAV of a fund is based on (a) Cash basis (b) Accrual basis (c) Income basis (d) None of the above 195.A contingent exit load fund (a) Charges no load (b) Charges load only on exit (c) Charges load both at entry and exit (d) Charges load if there is early redemption 196.Balanced funds have investments in (a) Equity (b) Debt (c) Both equity and debt (d) Short term instruments 197.An equity fund may have (a) Only a growth plan (b) Only a dividend plan (c) Both a growth and a dividend plan (d) All of the above 198.The following are benefits of mutual funds: (a) They do not carry risk (b) They are cost effective (c) Both a and b (d) None of the above 199.The following type of mortgage does not require possession to be handed over to the mortgagee (a) Simple mortgage (b) Usufructuary mortgage (c) English mortgage (d) All of the above

Financial Planning Academy

44

200.A combination of two or more investment types is called a (a) Usufructuary mortgage (b) Equitable mortgage (c) Anomalous mortgage (d) None of the above 201.The following mortgage is quite common among bankers (a) Simple mortgage (b) Mortgage by deposit of title deeds (c) Equitable mortgage (d) B and c 202.If the NPV of buy alternative is positive and NPV of incremental lease effect is negative, then the asset should be (a) Leased (b) Bought (c) Neither leased nor bought

203.If the NPV of both buy alternative and incremental lease effect is positive, the asset should be (a) Leased (b) Bought (c) Neither leased nor bought (d) (Incomplete information) 204.While considering the lease vs. buy decision, the incremental lease cashflow should be discounted at a discount rate, which is (a) Higher than buy alternative (b) Lower than buy alternative (c) Same as with buy alternative (d) Could be any of the above 205.The ______ is the lender of last resort (a) RBI (b) SEBI (c) Central Govt. (d) Commercial Banks 206.Prepayment of personal loan in a falling interest rate scenario will attract (a) No charges (b) Penal charges (c) Additional benefits (d) None of the above

Financial Planning Academy

45

207.For the rural sector, the refinancing arm is primarily (a) RBI (b) Regional rural banks (c) Commercial banks (d) Nabard 208.In hire-purchase, ownership of assets passes on to hirer (a) On commencement of the hire-purchase agreement (b) On completion of installments (c) On cash down payment (d) None of the above 209. Depreciation is charged, in a hire-purchase agreement, by the (a) Hirer (b) Hiree (c) Either, based on the agreement (d) Depreciation is disallowed 210.Entire hire-purchase installment is tax deductible as an expense (a) The above statement is true (b) The above statement is false (c) Only the interest expense is tax deductible (d) A and c 211.In a lease transaction, salvage value can be claimed by (a) Lessee (b) Lessor (c) Neither (d) Either 212.As opposed to installment sale, hire-purchase (a) Is more cumbersome (b) Charges more interest (c) Allows transfer of ownership when all installments are paid (d) All of the above 213.The security for a consumer loan is usually (a) A guarantee (b) A pledge agreement (c) Post-dated cheques (d) All of the above

Financial Planning Academy

46

214.When a loan is repaid by equated monthly installments, the interest (a) Goes on decreasing (b) Goes on increasing (c) Remains the same (d) None of the above 215.An emergency fund could be kept in the form of (a) Gold and silver (b) Saving bank account (c) Fixed deposits (d) b & c 216. Consumer credit is prompted by (a) Consumerism (b) Contingency (c) Childrens education (d) All of the above 217.Liquidity is needed (a) To avail discounts (b) To ensure smooth running of day to day life (c) To ensure peace of life (d) All of the above 218.An aberration in the normal yield curve would be when (a) Long term interest rates are below short term rates (b) Long term interest rates are above short term rates (c) Long term interest rates are same as short term rates (d) None of the above 219.The yield to maturity of a bond may also be called its (a) Interest rate (b) NPV (c) IRR (d) None of the above 220.The term structure of interest rates refers to (a) The interest rates in the longer term (b) The interest rates applicable to a term loan (c) The interest rates in the shorter term (d) Existence of different interest rates at different maturities

Financial Planning Academy

47

221.Short-term interest rates are lower usually because: (a) Greater risk is associated with the longer term (b) Short term investments imply higher liquidity (c) Investors are concerned about inflation in the longer term (d) All of the above 222.Deflation is (a) A boon (b) Good for developing economies (c) A phenomenon which has a number of negative aspects (d) Good for developed economies 223.Deflation is bad because (a) It reduces profits (b) Debt becomes excessive (c) a and b (d) None of the above 224.The principal amount of a trust is called (a) The fund (b) The corpus (c) The kitty (d) None of the above 225.A trust created by a will to manage the deceaseds assets is called (a) A revocable trust (b) A irrevocable trust (c) A testamentary trust (d) A charitable remainder unitrust 226.A probate is (a) A special type of trust (b) A joint trust (c) A trust formed under the Indian Trusts Act (d) Distribution of estate under court supervision 227.The minimum numbers of members of a society are (a) No minimum is prescribed (b) 10 (c) 20 (d) 7

Financial Planning Academy

48

228.The memorandum of a society contains (a) The names, addresses and occupations of its governors etc. (b) The property belonging to a society (c) Both a) and b) (d) None of the above 229.Professional associations are formed usually under the (a) Indian Trusts Act, 1882 (b) Societies Registration Act, 1860 (c) The Companies Act (d) None of the above 230.Corporations are formed under (a) The Companies Act (b) An Act of Parliament (c) Both a) and b) (d) None of the above 230.Example/s of national trade associations are (a) FICCI (b) CII (c) ASSOCHAM (d) All of the above 232.The following is not a way of taking title to property (a) By gifting (b) By exchange (c) By mortgage (d) None of the above 233.In the interest of investor protection, SEBI carries out (a) Market surveillance (b) Action against intermediaries (c) Action against erring companies (d) All of the above 234.Market surveillance activities of SEBI are mainly concerned with (a) Merchant bankers (b) Erring companies (c) Stock exchanges (d) Department of Company Affairs

Financial Planning Academy

49

235.Some of the market surveillance systems already in place by SEBI are (a) Daily price bands (b) Inspection of intermediaries (c) Both a and b (d) None of the above 236.Any advertisement soliciting fixed deposits from the public must contain (a) Paid up capital (b) Networth (c) Balance sheet size (d) None of the above 237.Deposits are normally allowed to be accepted as a multiple of (a) Paid up capital (b) Networth (c) Sales (d) None of the above 238.A small depositor is defined under the Companies Act as one who (a) Deposits less than Rs. 50,000 in a year (b) Deposits less than Rs. 50,000 in the last two years (c) Deposits less than Rs. 20,000 in a year (d) Deposits less than Rs. 10,000 in each of the last two years 239.An advisor stands in a fiduciary position to his client (a) True (b) False (c) True if there is a written contract (d) False if there is an agreement to the contrary 240.Fiduciary relationship requires (a) The use of the word trust (b) A written agreement (c) A constructive trusteeship (d) None of the above

Financial Planning Academy

50

PART- 5
1 Which of the following is true in regard to a Financial Planners liability? (a) A disclaimer removes all liability. (b) A principal advisor is liable for actions of representative. (c) Advice is distinguishable from a recommendation. (d) An advisor may be held liable for failure to predict economic changes 2 Which of the following is not a license/certificate according to present licensing regulations? (a) Stock-broker (b) Sub-broker (c) Insurance agent (d) All of the above are licensees/certificates 3 Which of the following tests apply to reasonable basis for recommendations? (1) Know your client (2) Obey Trade Practices Acts (3) Know relevant rules and regulations (4) Know your products (a) 1&2 (b) 2&3 (c) 3&4 (d) 1&4 4 The stage of the business cycle which is marked by increased consumer and investment spending, higher price levels and money wages and rising employment and national income is (a) Boom (b) Contraction (c) Recession (d) Recovery 5 Which of the following is not normally an influence upon short term interest rate? (a) Movements in the current account deficit (b) The trend of interest rates overseas (c) Fiscal policy (d) The rate of long term unemployment

Financial Planning Academy

51

6 When the government decides economic policy through the budget, it is (a) Monetary policy (b) Fiscal policy (c) Incomes policy (d) Exchange rate policy 7 Which of the following measures is most widely used as an indication of inflation? (a) GDP (b) WPI (c) CPI (d) None of these 8 Assuming all other things being equal, if the government increases the circulation of money, interest rates will: (a) Increase (b) Decrease (c) Stabilize (d) Remain unaffected 9 A life policy, which has no saving element or cash value, is: (a) Term insurance (b) Whole of life policy (c) Endowment policy (d) An annuity 10 What would be the main reason for a small investor using a mutual fund instead of direct investment in shares? (a) Lower market risk (b) Lower charges (c) Access to broad asset allocation (d) Higher returns 11 Which of the following are steps used in preparing a financial plan? 1 Goal setting 2 Identification of financial problems 3 Preparation of alternatives/recommendations 4 Implementation of agreed recommendations (a) (b) (c) (d) 1,2,3&4 1,2,&3 3&4 2,3&4

Financial Planning Academy

52

12 Which of the following applies at the time alternatives and recommendations are made? (a) The plan should be flexible enough to cope with the clients situation should it change in any way (b) The main focus should be on the performance of recommended investments (c) The client should be asked to think about plans for the future (d) All of the above 13 A statement of advice is not needed in providing limited financial planning advice. (a) There is no such thing known as a statement of advice. (b) The above statement is true. (c) The above statement is False (d) There is no such thing as limited financial planning advice, by definition financial planning is comprehensive. 14 Which of the following is true? (A) A professional financial planner provides only comprehensive financial advice to clients encompassing estate planning, insurance planning and risk management, income and expenditure (cash flow), retirement benefits, investment planning and taxation. (B) A professional financial planner may provide limited advice, if he discloses the fact at the outset to the client. (a) (b) (c) (d) Both A &B are false A is true but B is false. A is false but B is true. A is true provided, the financial planner is a CFP

15 A professional financial planner is one who (a) Takes pride in his/her work (b) Is committed to quality (c) Is dedicated to the interest of the client (d) All of the above 16 According to the FPSB rules of professional conduct, disclosures regarding compensation needs to be made only at the time of establishing the relationship with a new client. (a) True (b) False (c) Sources of compensation need not be disclosed. (d) Need to be disclosed whenever there is a change in status

Financial Planning Academy

53

17 Any personal information about the client may not be used by the financial planner except. (a) Unprofessional (b) Advertising (c) Smart thinking (d) Waste of time 18 A CFP certificant shall act in a ------- capacity as regards clients funds (a) Trustee (b) Beneficiary (c) Fiduciary (d) Professional 19 In periods of inflation, nominal rates are ---------than real interest rates (a) Higher (b) Lower (c) Equal (d) None of these 20 Legislation risk ---------- minimized by diversifying investments across sectors. (a) Can be (b) Cant be (c) Legislation does not have any impact (d) None of these 21 --------is entitled to claim depreciation but not a --------(a) Hirer, lessee (b) Lessees, hirer (c) Lessor, hirer (d) Lesses, lessor (Hirer will become owner after certain number of years, lessee is tenant) 22 A bond with a coupon rate of 8% is available at its face value of Rs 1000. If the market rate of return on an investment with similar risk goes down to 6%,the bond price will become (a) 1000 (b) 750 (c) 1333 (d) None of these.

Financial Planning Academy

54

23 A bond with a coupon rate of 10% is available at Rs 1250.The face value of the bond is RS 1000,the effective yield on the bond is (a) 10% (b) 8% (c) 12% (d) None of these 24 ALC In the context of cash flow planning and budgeting refers to (a) Average living costs (b) Annual living cashflow (c) Annual living costs (d) None of these 25 If the inflation rate is 3% and the tax rate is 40%, the required rate to maintain the value of an investment is (a) 4% (b) 5% (c) 6% (d) 7% (Required rate is =Inflation/(1-tax rate)=3/.60=5%) 26 The code of ethic of integrity details conduct rules relating to (a) Members compensation (b) Promotional activities (c) Clients funds (d) B & C 27 The code of ethics of fairness requires (a) That compensation of a financial planner be fair and reasonable (b) That partners in a financial planning firm should act in good faith (c) That a member may provide reference of other clients to establish a relationship (d) All of the above 28 An FPSB member is required to keep all office/client records for a period of ---years (a) Four (b) Seven (c) Eight (d) Five 29 Professional responsibility is based on (a) Contractual obligation (b) A duty of care to the client (c) Fiduciary relationship (d) All of the above

Financial Planning Academy

55

30 A contract may be discharged by (a) Performance (b) Impossibility of performance (c) Breach (d) All of these 31 Mr. A plans to invest Rs 10000 today for a period of 4 years. If interest rate is 10% p.a., how much income per year should he receive to recover his investment? (a) Rs 3155 (b) Rs 2500 (c) Rs 2800 (d) Rs 3000 32 The following type of mortgage does not require possession to be handed over to the mortgagee. (a) Simple mortgage (b) Usufructuary mortgage (c) English mortgage (d) All of the above 33 A combination of two or more mortgage types is called a (a) Usufructuary mortgage (b) Equitable mortgage (c) Anomalous mortgage (d) None of these 34 The yield to maturity of a bond may also be called its (a) Interest rate (b) NPV (c) IRR (d) None of these 35 As part of their job, financial planners have to predict future economic indicators which may impact clients (a) True (b) False (c) It is true in cases of economic turmoil (d) The statement is true for CFP Certificants 36 Micro-economics refers to the study at (a) National level (b) Level of the firm (c) Personal level (d) b & c

Financial Planning Academy

56

37 High level of employment, faltering business, slackening rate of investment activity, higher costs for business firms are indicators of which stage of the business cycle. (a) Contraction (b) Recession (c) Recovery (d) Boom 38 Asset allocation strategy is guided by (a) Portfolio diversification (b) Clients risk profile (c) Objectives of the client (d) All of the above 39 The following is not an essential component of a financial plan (a) Executive summary (b) Financial planning strategy (c) Letter of engagement (d) Summary of services provided 40 A bank has offered to you an annuity of Rs 1800 for 10 years . If you invest Rs 12000/today the effective rate of return in this case is (a) 6.67% (b) 8% (c) 8.15% (d) 8.67% ( Pv= -12000, 1800=pmt, 10= n, compute I) 41 In 2002, X Ltd has issued bonds of Rs 10000 each due in 2012 with a 14%p.a. coupon rate payable at the end of each year during the life of the bond. The present value of the bond is---------------- if the Required rate of return is 14% (a) Rs 10000 (b) Rs 9500 (c) Rs 2758 (d) Rs 15000

42 In an American option , exercise is possible (a) Only at the date of expiry of option (b) At any time before the expiry of option (c) At any time until the expiry of the option (d) None of these

Financial Planning Academy

57

43 In a PUT option .there is a (a) Right but not the obligation to sell a specified asset at a specified price or within a specified price. (b) Right but not the obligation to buy a specified asset at a specified price, at OR WITHIN A SPECIFIED PRICE. (c) Obligation but not the right to sell a specified asset at a specified price or within a specified price. (d) Obligation but not the right to buy a specified asset at a specified price at or within a specified price 44 Integrity implies (a) Members shall provide financial planning services in a fair and irrevocable manner (b) Members shall use high standards of honesty in conducting their financial planning business. (c) Members shall disclose to their client any limitation on their inability to provide objective financial planning services (d) All of these 45 Which of the following is not an essential components of a written financial plan? (a) Statement of current situation (b) Financial plan summary (c) Services, fee and commission etc (d) Projections (e) All of the above are essential components 46 A financial plan should be reviewed in the light of (a) Micro and Macro level changes (b) Micro and not Macro level changes] (c) Macro and not Micro level changes (d) Generally the financial plans do not require review 47 Which of the following risks falls under the category of Pure risk that people face. (a) Loss of use of property (b) Risk of injury (c) Loss arising from negligence of third party (d) Risk of loss of income through incapacity to work for some extended work (e) All of the above

Financial Planning Academy

58

48 Which of the following is generally excluded in goal setting step in financial plan development? (a) Cash flow analysis (b) Determining financial objectives (c) An assessment of priorities (d) Establishing basic needs 49 A CFP Certificant will cooperate with FPSB in all respects of an investigation is a requirement of code of ethic of (a) Professionalism (b) Diligence (c) Compliance (d) Confidentiality 50 A Rs 1000 bond has a 6% annual coupon and is due in two years. Its value in todays market is Rs 900.The Yield to Maturity of this bond is (a) 9.90% (b) 10.40% (c) 11.90% (d) 11.00% (e) None of these

Financial Planning Academy

59

PART- 6
QUESTION BANK FOR PRACTICE- FINANCIAL MATHEMATICS 1 You save Rs 2000 a year for 5 years, and Rs 3000 a year for 10 years thereafter. How much these savings will grow into? Assume a rate of interest of 10% (Rs 79482.38) 2 Mr X receives a PF amount of Rs 1,00,000. He deposits it in a bank which pays 10% interest. If he withdraws annually Rs 20,000 for how long he can do so? (7.27 years) 3 You want to borrow Rs 15,00,000 to buy a flat. You approach a housing company which charges 13% interest. You can pay Rs 2,00,000 per year towards loan amortisation. What should be the maturity period of the loan? (30.18 years) 4 If you deposit RS 5000 today at 12% rate of interest, in how many years will this amount grow to Rs 1,60,000? (30.58 years) 5 A finance company offers to give Rs 8000 after 12 years in return for Rs 1000 deposited today. Figure out the approximate interest offered. (18.92%) 6 A finance company advertises that it will pay a lump sum of Rs 10000 at the end of 6 years to investors who deposit annually Rs 1000.What interest rate is implicit in this? (20.28%) 7 At the time of his retirement , Mr Jingo is given a choice between two alternatives: (a) An anuual pension of Rs 10,000 as long as he lives and (b) A lump sum amount of Rs 50,000. If Mr Jingo expects to live for 15 years and the interest rate is 15%, which option appears more attractive? (option (a) is better PV Rs 58473.70) 8 Mr. X deposits Rs.1,00,000 in a bank which pays 10% interest. How much can he withdraw annually for a period of 30 years. Assume that at the end of 30 years the amount deposited will whittle down to zero. (Rs10607.92 Pmt) 9 What is the present value of Rs 2,000 receivable annually for 30 years? The first receipt occurs after 10 years and the discount rate is 10%. (7268.96 PV) 10 Phoenix company borrows Rs 5,00,000 at an interest rate of 14%. The loan is to be repaid in 4 equal annual installments payable at the end of each of the next 4 years . Prepare the loan installment amount? (Rs 171602 Pmt per annum)

Financial Planning Academy

60

Part - 7
1 There is a 25 years old male with no dependants and nominal income. His topmost priority should be: a) Retirement Planning (b) Income protection (c ) Life Insurance (d) Critical Illness 2 You invest Rs.50,000 in a commercial real estate property and expect to get Rs.180000 (after selling expenses etc) when you sell in 8 years. What is your expected rate of return on this property? (a) 17.63% (b) 18.36% (c) 17.36% (d) 17.01% 3 What is the value of stock in a company that pays out Rs.1.50 per share in dividends and expects these dividends to grow at 6% per annum forever. Investors require a return of 13% on stocks of equivalent risk. (a) Rs.20.43 (b) Rs.22.70 (c) Rs.20.67 (d) Rs.21.67 (Hint: P=D1/r-g where D1 is=D0(1+g),r =required return, g = growth rate D0=dividend for current year) 4 The Rule of 72 says that if you earn 8% per year, your money will double in -----years. (a) 12 (b) 6 (c) 8 (d) 9 (e) 72 5 The term structure of interest rates plots yield verses: (a) All maturities of a particular risk class of bonds at a point in time. (b) A single maturity of a particular risk class of bonds of time (c) All maturities of one category of bonds at a point in time (d) All risk classes of bonds at a point in time

Financial Planning Academy

61

6 Mr. Gupta purchased a 2 year bond bearing a 12% coupon rate. He purchased the bond at par ( Rs.1000). If rates fall to 9% what will be the new price of the bond? (a) 1053 (b) 1136 (c) 1193 (d) 900 7 Beta --------------------(a) Is a measure of firm specific risk (b) Is a measure of market risk (c) Is a measure of total risk (d) Is calculated by regressing the market return on the historical market divided yield (e) All of the above are true 8 Firm specific risk is also called --------(a) Market risk (b) Macro risk (c) Undiversifiable risk (d) Non systematic risk (e) Unavoidable risk 9 Six years ago ABC Ltd issued 10 years bond with a coupon rate of 11%. If the investors required rate of return (IRR) on these bonds is 9%, the bonds will be priced at (a) Below par (b) Above par (c) Above and below par (d) At par 10 Which of the following best describe a debenture? (a) A long term corporate promissory note (b) An investment in the debt of another corporate entity (c) A long term corporate debt obligation with a claim against securities rather than against physical assets (d) A corporate debt obligation that allows the holder to repurchase the security at specified dates before maturity. (e) Unsecured corporate debt 11 The new Senior citizens Savings Bond scheme offers ---------------% interest. (a) 8.5 (b) 9 (c) 9.25 (d) 8.75

Financial Planning Academy

62

12 (a) (b) (c) (d)

The maximum amount that can be invested in Public Provident Fund is Rs.----------70000 60000 80000 90000

13 ------------ is regulated by The Reserve Bank Of India . (A) Bank deposit rate (B) Bank lending rate (C) Certificate of deposit rates (a) A (b) B (c) C (d) None of these 14 The recent Union Budget exempts---------------- from Long term Capital Gains tax subject to certain conditions. (a) Equity shares (b) Debt mutual funds (c) Property (d) Gold 15 (a) (b) (c) The deduction from Gross Total Income available u/s 80C is Rs.------------100000 145000 150000

16 -------------is /are governed by SEBI. (a) Mutual Funds (b) Stock brokers (c) Portfolio managers (d) All of the above 17 As far as employment and production are concerned which of the following industries are more affected by recession? A. Capital goods B. Consumer durable goods C. Consumer non durable goods D. Services (a) a and c only (b) a and b only (c) b and c only (d) c and d only (e) b and d only

Financial Planning Academy

63

18 Which combination of the following statements is true regarding the investment strategy known as rupee cost averaging? 1) Invests the same rupee amount each month over a period of time. 2) Purchase the same number of shares each month over a period of time. 3) Lowers average cost per share over a period of time (assuming share price fluctuations) 4) Invests the same rupee amount each month to protect the investment from loss of capital (a) (1) & (2) only (b) (1) & (3) only (c) (2) & (3) only (d) (2) & (4) only (e) (1) (2) (3) & (4) only 19 Which of the following are fundamental regarding characteristic of insurance? 1) Probability of loss 2) Law of large numbers 3) Transfer of risk from individual to group 4) Insurance is a form of speculation (a) 1 and 2 only (b) 1 2 and 4 only (c) 1 2 and 3 only (d) 4 only (e) 1 2 3 and 4 20 Mr. X invests in a limited partnership, which require an initial outlay of Rs.9400 today. At the end of years 1 through 5 he will receive the after tax cash flows of shown below. Year Cash Flows (Rs.) 0 (-9400) cfj 1 600 2 2300 3 2200 4 6800 5 9400 The after tax IRR of this investment is : (a) 23.34% (b) 28.00% (c) 22.94% (d) 24.25%

Financial Planning Academy

64

21 A client with a large well diversified common stock portfolio expresses concern about a possible market decline. However he does not want to incur the cost of selling a portion of their holdings nor the risk of mistiming the market. The possible strategy for him would be (a) Buy an index call option (b) Sell an index call option (c) Buy an index put option (d) Sell an index put option (e) He cannot protect against the decline with these options 22 Arrange the following financial planning functions into the logical order in which a professional financial planner performs these functions. 1. Interview clients, Identify preliminary goals 2. Monitor financial plans 3. Prepare financial plan 4. Implement financial strategies, plans and products 5. Collect, analyze, and evaluate client data (a) 1 3 5 4 2 (b) 5 1 3 2 4 (c) 1 5 4 3 2 (d) 1 5 3 4 2 (e) 1 4 5 3 2 23 The standard deviation of the returns of a portfolio of securities will be --------the weighted average of the standard deviation of returns of the individual component securities (a) Equal to (b) Less than (c) Greater than (d) Less than or equal to (depending on the correlation between securities) (e) Less than, equal to or greater than (depending on the correlation between securities) 24 According to the fundamental analysis, which phrase best describes the intrinsic value of a share of common stock? (a) The par value of the common stock (b) The book value of the common stock (c) The liquidating value of the firm on a per share basis (d) The stocks current price in an inefficient market (e) The discounted value of all future dividends

Financial Planning Academy

65

25 If the market risk premium were to increase, the value of common stock(everything else being equal) would (a) Not change because this does not affect stock values (b) Increase in order to compensate the investor for increased risk (c) Increase due to higher risk free rates (d) Decrease in order to compensate the investor for increased risk (e) Decrease due to lower risk free rates ( risk is related to return, therefore price will decrease to provide compensation to new investors, otherwise with increased risk nobody will be interested to purchase this share) 26 In computing portfolio performance, the Sharpe index uses---------, while the Treynor Index uses-------------------- for the risk measure. 1. Standard deviation 2. Variance 3. Correlation coefficient 4. Coefficient of variation 5. Beta (a) 5 1 (b) 1 3 (c) 1 4 (d) 1 5 (e) 2 5 27 Which of the following is non-diversifiable risk? 1. Business risk 2. Management risk 3. Company or industry risk 4. Market risk 5. Interest rate risk 6. Purchasing power risk (a) (b) (c) (d) (e) 456 123 562 134 146

Financial Planning Academy

66

28 American depository receipts (ADR) are used to 1. Finance foreign exports 2. Eliminate currency risk 3. Sell U.S. securities in overseas market 4. Trade foreign securities in U.S. markets (a) 1 3 (b) 1 4 (c) 2 4 (d) 4 (e) 1 2 4 29 If R is the real return, r is the portfolio return and I is the rate of inflation, then the formula for calculation of R is: (a) R= r-1 (b) R=1-(r+1)/(r-1) (c) R=1+(R+1)/(r-1) (d) R={(1+r)/(1+I)}-1 30 As a Certified Financial Planner, you have selected a product which will be most suitable to the clients requirement, but the client is not able to comprehend the product. In the circumstances as per code of ethics: (a) You have not violated the code as the product suits the best interest of the client (b) Violated the code as the client is not able to comprehend the product (c) The policy can be cancelled and fresh policy can be issued 31 The Convertible Term Assurance policy means: (a) The policy can be surrendered after sometimes for cash value (b) The policy can be converted into endowment plan (c) The policy can be cancelled and fresh policy can be issued 32 You are likely to receive Rs.85000 and Rs.91000 at the end of 19 and 21 years and if the discount rate is 6%, what is the present value? (a) 54862 (b) 54678 (c) 54863 (d) 51234 (28093.61 and 26768.14 respectively) 33 In the regime of soft interest, which of the following actions taken by RBI will not increase the bond prices significantly? (a) Selling of Treasury Bills at low rates (b) Move to privatize and improve tax collections (c) Deficit financing

Financial Planning Academy

67

34 You wish to save for your daughters education, the present cost of which is around Rs.240000 and is expected to grow every year at the rate of 7%. If your daughter is 10 years old and is likely to be in the college in another 8 years time, what is the amount of investment to be made if it is likely to earn 11% return? (a) 178936 (b) 179836 (c) 173896 (d) 178930 ( calculate FV of Rs.240000 @7% for 8 years =412364, then calculate PV FOR THE SAME) 35 While choosing a product for your client which involves investment predominantly in Government securities, you make the following remarks: 1. The portfolio will have low risk 2. The rate of return earned by the fund in the past is 8% In the light of AFP Code of ethics, which of the following are incorrect as it is violative (a) Statement A (b) Statement B (c) Both (d) None 36 If you are managing a balanced portfolio and frequent changes are made in allocation of assets, it will result in (a) Increase in transaction costs (b) Decrease in transaction costs 37 The difference between Estate planning of a salaried employee and self employed person: (a) Both have to provide for dependants (b) Have to write wills (c) Take insurance policies (d) Employee benefits 38 How many years will it take fort a sum of Rs.10000 to double if the rate of return is is 9% p.a. (a) 9.5 (b) 8.5 (c) 10 (d) 9 (e) 8

Financial Planning Academy

68

39 If the post tax rate of return on an investment is 9% and the inflation rate is 5%, the real rate of return is (a) 3.81% (b) 3.0% (c) 2.86% (d) 2.74% 40 Radha and Ram are co-applicants of a mortgaged house. They are on the verge of a divorce. The Housing Finance company will--------(a) Not interfere as long as the EMIs are being paid on time (b) Repossess the house after divorce (c) Insist on the house being transferred to one of them (d) Mediate reconciliation between the couple (e) Increase the interest rate in order to compensate for the increased risk 41 Domestic GOI bond holders (holding them up to maturity) have to deal with ---------risk. (a) Volatility (b) Default (c) Inflation (d) Price (e) Currency 42 Refinancing is -------------------(a) Borrowing at lower cost in order to pay off higher cost debt (b) Repaying debt by selling off assets (c) Lending at a higher rate of interest (d) Securitizing your receivables (e) None of these 43 -------------------Asset allocation is not a text book Asset Allocation Model. (a) Tactical (b) Discretionary (c) Strategic (d) All of the above (e) None of the above

Financial Planning Academy

69

44 Ram and Shyam approach you to be their financial planner. Their funds are limited and their needs are many. Some of their needs are 1. To start an investment plan for funding their child education 2. To set up a Testamentary trust for their child 3. To set up a contingency fund amounting to 6 months of living expenses 4. To start saving for retirement 5. To purchase life and health insurance (a) (b) (c) (d) 35142 45231 24513 25134

45 For a nominal interest rate of 6% payable monthly, quarterly and semi annually, the effective rates respectively would be--------------(a) 6.04 6.02 6.01 (b) 6.16 6.13 6.09 (c) 6.10 6.07 6.03 (d) 6.11 6.08 6.06 46 A 10 year 9% bond (face value of Rs.100,interest paid annually) maturing 3 years from today is available at a YTM of 5.8%. Therefore the current price is ---------(a) 152.50 (b) 154.10 (c) 108.58 (d) 107.00 47 Aman wants to purchase a car 5 years from now. His investments are currently worth Rs.50000 and he intends to contribute Rs.10000 at the beginning of every six months period to fund his purchase. Assuming that the annual investment rate of return is 8% compounded semi-annually, what will be the value of the investment in 5 years time? (a) 198875 (b) 195555 (c) 197240 48 Neelu wants to accumulate Rs.150000 in 3 years time for a one month trip to Europe. Assuming she can get an 8% annual return on her investments, compounded quarterly, how much must she invest today in order to achieve her goal? (a) 117591 (b) 119487 (c) 118274

Financial Planning Academy

70

49 Rohan invested Rs.420000 for 7 years @7% where it was compounded annually for the first 5 years and quarterly for the last 2 years. What did he receive on maturity? (a) 676774 (b) 776774 (c) 931095 (d) 609870 50 A contract with a minor is (a) Is voidable at the option of either party (b) Is absolutely void (c) May be admissible in special cases (d) None of these 51 Inchoate instruments are (a) Currency notes in circulation (b) Bills of exchange (c) Signed blank cheques (d) Signed blank stamp papers 52 Tort does not cover (a) Civil wrong (b) Breach of contract (c) Defamation (d) None of these 53 An investor expects a perpetual sum of Rs.50000 annually from his investment. What is the present value of this perpetuity if rate is 10% p.a. (a) Rs.15 Lakhs (b) Rs.5 Lakhs (c) Rs.10 Lakhs (d) None of these 54 A combination of two or more mortgages is called (a) Usufructuary mortgage (b) Equitable mortgage (c) Anomalous mortgage (d) None of these 55 Liquidity is needed (a) To avail discounts (b) To ensure smooth running of day to day life (c) To ensure peace of mind (d) All of the above

Financial Planning Academy

71

56 Deflation is (a) A boon (b) Good for developing economies (c) A phenomenon which has a number of negative aspects (d) Good for developed economies 57 A probate is (a) A special type of trust (b) A joint trust (c) A trust formed under the Indian Trusts Act (d) Distribution of estate under court supervision 58 (a) (b) (c) (d) A small depositor is defined under the Companies Act as one who Deposits less than Rs.50000 in a year Deposits less than Rs.50000 in the last two years Deposits less than Rs.20000 in a year Deposits less than Rs.10000 in each of the two years

59 If a planner does not receive sufficient and relevant information from a client he/she should (a) Terminate the relationship (b) Give restricted (limited) advice (c) Go ahead but give a disclaimer disclaiming all responsibility (d) Either a or b 60 The client questionnaire records quantitative data and-------------------records qualitative data (a) The planners mind (b) File notes (c) The same client questionnaire (d) A separate client questionnaire 61 A strategic review of a clients situation is required in case of (a) Macro level changes (b) Micro level changes (c) Neither (d) Both 62 A letter of engagement is : (a) A legal document (b) A requirement of the AFP s rules of professional conduct (c) Essential for client-planner relationship (d) Like a memorandum of understanding

Financial Planning Academy

72

63 Which of the following can generally be excluded from the definition of securities
for general purposes? (Is it from the point of view of Non-tradable securities) (a) A prescribed interest in a unit trust (b) An option contract (c) Interest in a business undertaking (d) Bank certificate of deposit 64 A client of a financial planner who sustains losses and claims the financial planner is at fault, may be able to base a claim under: (a) Law of contract only (b) Law of tort only (c) The law of contract and the law of tort (d) Criminal law 65 The goal setting step in financial plan development would generally not include which of the following? (a) Determining objectives (b) Cash flow analysis (c) Assessing priorities (d) Establishing basic needs 66 Client risk profiling by questionnaires is best used (a) Rather than indirect method s of assessing client risk (b) As an equally valid method of assessing risk compared to more indirect methods (c) As a supplementary adjunct to other means of assessing risk (d) As the only way of assessing the risk profile for the client 67 The principle of subrogation means that: (a) An insurer may bring an action against a third party in the name of the insured after satisfying a claim (b) An insured may not take legal action in respect of a matter which is the subject of a claim (c) An insurer undertakes to take legal action on behalf of its insured and to pay to the insured the net proceeds of such legal action (d) An insurer which has satisfied a claim may recover from the insurer of a third party who admits liability

Financial Planning Academy

73

68 The distinction between a futures contract and an options contract is : (a) A futures contract imposes an obligation on the buyer and the seller while an options contract imposes an obligation on the seller only and confers a right on the buyer (b) A future contract imposes an obligation on the buyer and the seller while an option contract confers a right on both buyer and seller but not the obligation (c) A futures contract confers a right on the buyer and seller while an options contract imposes an obligation on the seller only and confers a right on the buyer (d) A futures contract confers a right on the buyer and seller while the options contract confers an obligation on both buyer and seller 69 A firm purchases a machinery for Rs.800000 by making a down payment of Rs.150000 and the remainder in equal annual installments of Rs.150000 for 6 years. How much is the rate of interest that the firm is paying? (a) 12.02% (b) 10.17% (c) 10.85% (d) 11.00% 70 Amount received from the surrender of annuity plan or amount received as pension from the annuity plan by the assessee or his nominee shall be (a) Exempt (b) Taxable (c) Exempt upto limit balance payable (d) None of the above 71 Deductions under section 80D in respect of medical insurance premium is allowed to (a) Any assessee (b) An individual or HUF (c) Individual or HUF who is resident in India (d) Individual only 72 Sudha is optimistic about the long term growth of her share. However the share price currently priced at Rs.58, has made a sharp advance in the last week and she wants to lock in minimum price in case the shares drop. What might Sudha do? (a) Buy Rs.55 call options (b) Sell Rs.55 call options (c) Buy Rs.55 put options (d) Sell Rs.55 put options

Financial Planning Academy

74

73 If the client needs to accumulate wealth but is risk averse, which of the following is the most crucial action the planner needs to take to have the client achieve the goal of wealth accumulation? Advise investing the clients current assets (a) In the products which will bring the highest return to the client regardless of risk. (b) In products which produce high income for the client because fixed income products are generally safe (c) In diversified mutual funds because of the protection which diversity provides (d) After determining the clients risk tolerance. (e) In 100% cash equivalents in the portfolio because most software programs recommend this safe approach 74 The investment portfolio for a defined benefit retirement plan has declined in value during a year in which most financial market investments have incurred losses. Which one of the following entities would be impacted most by this decline in portfolio value? (a) Individual participants in the plan (b) Company sponsoring the plan (c) Investment banker handling the plan (d) Plan underwriters 75 Which of the following would result in the largest increase in the price of a diversified common stock mutual fund? (a) Unexpected inflation (b) Expected dividend increases (c) Unexpected corporate earnings growth (d) Expected increase in the prime interest rate 76 Compute liquidity Ratio. Cash expenses per month=30000, Salary=60000, Bank savings a/c 150000 (a) 4 (b) 5 (c) 2 (d) 6 77 A person who declare will after the death of a person is called (a) Testator (b) Executor (c) Testate (d) Deceased

Financial Planning Academy

75

78 Your client, you have purchased shares of a company, now market fall, As a financial planner what would you do instantly? (a) You will explain the logic of your decision to client (b) You will buy more shares (c) You will sell out those shares (d) No step you will take 79 If the loan of Rs.3,00,000 is to be repaid in 6 annual installments with a coupon rate of 12% p.a. then the equated annual; installment will be: (a) Rs.71967 (b) Rs.72967 (c) Rs.74005 (d) Rs.75995 (e) Rs.76004 80 A person took a loan of Rs.10,000 on Jan 1,2003. At the end of every month he has to pay Rs.1000 for 12 months so that his loan will be totally repaid by December 31,2003. The implied interest rate per annum is (approx) (a) 20% (b) 25% (c) 28% (d) 35% (e) 42.% 81 If the annual cash flow for a bond is Rs.200, the present value of the cash flows if the inflows continue for 5 years at a required rate of 11% is (a) Rs.639 (b) Rs.739 (c) Rs.839 (d) Rs.869 (e) Rs.939 82 An income stream provides Rs.2000 for first three years and Rs.3000 for next three years, if interest rate is 14%, then the present value of income stream is: (a) Rs.8650.85 (b) Rs.8860.50 (c) Rs.9403.20 (d) Rs.9624.25 (e) Rs.9344.00

Financial Planning Academy

76

83 Mr. Suresh deposited Rs.1000 every month in a bank for five years, if the interest rate is 12% p.a. compounded monthly, then the accumulated amount he will get after 5 years is: (a) Rs.44955 (b) Rs.67200 (c) Rs.81600 (d) Rs.81670 (e) Rs.96000 84 XYZ Ltd had taken a loan of Rs.500 Lakh from a bank. The loan is to be repaid in ten equal annual installments. If the annual interest rate is 16%, then each installments is (a) Rs.102.78 Lakh (b) Rs.103.46 Lakh (c) Rs.111.43 Lakh (d) Rs.113.50 Lakh (e) Rs.132.13 Lakh 85 Mr. Rahul expects to receive from his friend an amount of Rs.2000 per annum for 10 years. If his required rate of return is 12% p.a. What is the present value of these cash inflows? (a) Rs.20000 (b) Rs.15000 (c) Rs.11300 (d) Rs.10500 (e) Rs.10000 86 If a borrower promises to pay Rs.20000 eight years from now in return for a loan Rs.12550 today, what interest rate is being offered? (a) 1.59% (b) 5.00% (c) 6.00% (d) 7.00% (e) 7.42% 87 Rs.10000 is being borrowed to be paid in four equal annual payments with 8% interest. Approximately, how much principal is amortized with the first payment? (a) Rs.800 (b) Rs.2219 (c) Rs.2500 (d) Rs.3281 (e) Rs.3300

Financial Planning Academy

77

88 The future value of a regular annuity of Re.1.00 earning a rate of interest of 12% p.a. for 5 years is equal to (a) Rs.6.250 (b) Rs.6.353 (c) Rs.6.425 (d) Rs.6.538 (e) Rs.6.625 89 The amount that has to be invested at the end of every year for a period of 6 years at a rate of interest of 15% in order to accumulate Rs.1000 at the end of 6 years is equal to (a) Rs.112.42 (b) Rs.114.24 (c) Rs.114.42 (d) Rs.112.44 (e) Rs.112.24 90 Money has time value because (a) The individuals prefer future consumption to present consumption (b) A rupee today is worth more than a rupee tomorrow in terms of its purchasing power (c) A rupee today can be productively deployed to generate real returns tomorrow (d) The nominal returns on investments are always more than inflation thereby ensuring real returns to the investors (e) Both b and c above 91 The present value of Rs.10,00,000 receivable after 60 years , at a discount rate of 10% is (a) Rs.3284 (b) Rs.6898 (c) Rs.18649 (d) Rs.39440 (e) Rs.48376 92 If the interest rate is 12% p.a., the amount to be invested today to earn an annuity of Rs.1000 for five years commencing from the end of first year is (a) Rs.6353 (b) Rs.5672 (c) Rs.4037 (d) Rs.3605 (e) Rs.3037

Financial Planning Academy

78

93 M/S Lee Ltd. Has Rs.10,00,000 worth of debentures to be redeemed after five years from now. If the interest rate is 14% p.a., the amount that has to be invested every year in a sinking fund to retire the above bonds is (a) Rs.661010 (b) Rs.343308 (c) Rs.291284 (d) Rs.151284 (e) Rs.134330 94 The amount to be invested today to earn an annuity of Rs.1000 for five years commencing from the end of two years from today if the interest rate is 12% per annum is (a) Rs.3218 (b) Rs.5993 (c) Rs.2874 (d) Rs.3873 (e) Rs.4873 95 Calculate the price of the par value Rs.100, the coupon rate is 13% of 8-year bond on which interest is paid semi-annually. The required return on this bond is 14%. (a) Rs95.30 (b) Rs.96.00 (c) Rs.96.99 (d) Rs97.02 96 A Rs.100 par value bond bears a coupon rate of 14% and matures after 5 years. Interest is paid semi-annually. Compute the value of the bond if the required rate of return is 16% (a) 94.50 (b) 93.29 (c) 95.25 (d) 93.90 97 A 10-year annual annuity has a yield of 9%. What is the duration? (a) 4.798 (b) 4.980 (c) 5.235 (d) 5.000 (Hint: 1+yield/yield-number of payments/(1+yield)no of payments-1) page no-337 of Prasanna Chandra-bond valuation

Financial Planning Academy

79

98 Compute the current yield of a 10 year, 12% coupon bond with a par value of Rs.1000 and selling for Rs.950 . (a) 12.63% (b) 13.50% (c) 12.00% (d) 13.25% 99 A Rs.1000 par value bond, carrying a coupon rate of 9%, maturing after 8 years is currently selling at Rs.800. What is the YTM of the bond? (a) 13.2% (b) 14.15% (c) 13.58% (d) 14.01% 100 The IDBI deep discount bond offers investors Rs.200000 after 25 years, for an initial investment of Rs.5000. The interest rate implied in the offer is (a) 14.81% (b) 15.00% (c) 15.90% (d) 16.50% (e) Not possible to determine from the given data

Financial Planning Academy

80

Part - 8
1 A 10 percent coupon bond has a maturity of 12 years. It pays interest semi-annually. Its yield to maturity is 4% per half year period. What is duration? (a) 7.56 years (b) 7.90 years 1+y - (1+y)+T(c-y) (c) 8.20 years y c[(1+y)t 1]+y (d) 8.15 years 2 A 10 year annual annuity has a yield of 9%. What is its duration? (a) 4.508 years (b) 4.798 years (c) 5.350 years (d) 5.205 years 3 A well to do individual who has his needs taken care of and is well into his retirement should invest into (a) 90% short term debt, 10% cash (b) 90% long term debt, 10% cash (c) 60% equity,30% debt,10% cash (d) 50% equity,40% debt,10% cash 4. There is a 35 year old man with a wife and 2 children aged 6 and 9 years. Their topmost priority should be (a) Retirement Planning (b) Income Protection (c) Life Insurance (d) Critical illness Insurance 5. There is a 22 year old male with no dependants and nominal income. His topmost priority should be: (a) Retirement Planning (b) Income protection (c) Life Insurance (d) Critical Illness 6 Observing an appropriate standard of care as a professional means that: (a) A mistake or error constitute a breach of standard (b) Standards are defined by the relevant professional body (c) Conformity with current professional standards is always sufficient (d) The professional is expected to demonstrate the standard of the ordinary person professing to have the skill

Financial Planning Academy

81

7 In simple term the legal term negligence is : (a) Not acting as a reasonable person (b) A basis for liability (c) A breach of contract (d) The same as tort

8 Which of the following are not securities ( is it tradable security)


(a) A prescribed interest in a unit trust (b) An option contract (c) Interest in a business undertaking (d) Bank certificates of deposit 9 You meet an analyst who believes that a share has specific intrinsic value that can be determined at any point in time. She states that although the market price will differ from this intrinsic value, it will eventually move back to this value. Such an analyst is : (a) Technical analyst (b) Fundamental analyst (c) Chartist (d) Speculative analyst 10 Performance measurement by benchmarking normally refers to comparing performance of a fund to: (a) The performance achieved by comparable investment funds (b) The result of key economic data (c) The results achieved by key investment indexes (d) The results achieved by recognized superannuation funds 11 A research house wishes to carry out qualitative research. Which of the following is an example of qualitative research? (a) Collecting data on the historic performance of funds (b) Comparing the returns from funds (c) Comparing the strengths and weaknesses of management (d) Ranking funds according to assets 12 A research house wishes to carry out quantitative research. Which of the following is an example of quantitative research? (a) Comparison of relative performance against fund managers (b) Comparison of product features (c) Consideration of quality of assets (d) Quality and experience of investment team

Financial Planning Academy

82

13 All recommendations concerning the financial affairs of a client should be presented in writing because: A. It is mandated under Companys law B. It is either mandated or regarded as best practice under the FPSB Code of Ethics and Rules of Professional Conduct C. It gives the client the necessary time to fully consider the recommendations D. It provides substantial protection from common law claims of negligence (a) c and d only (b) b c and d only (c) a b and c only (d) a b c and d only 14 The fixed deposit scheme of a bank offers 10% pa interest for a three year deposit. If the compounding is done semi-annually, then effective annual interest rate is: (a) 10.00% (b) 10.25% (c) 10.38% (d) 10.50% 15 The difference between the effective rate of return of a bond with a coupon rate of 12% when compounding monthly and quarterly is: (a) 0.03 (b) 0.10 (c) 0.13 (d) 0.19 (e) 0.25 16 Mr. X has taken a loan of Rs.6,00,000 from a housing finance company on an interest rate of 12% p.a. The terms of payment are Rs.100000 cash down and balance in 10 equal annual installments. Calculate the amount of annual installment? (a) 106190 (b) 88492 (c) 95496 (d) 75000 17 Mr. A wishes to purchase a house with the help of loan amount. He approaches a housing finance company and explains to them that his paying capacity is Rs.3540 per month. Rate of interest to be charged by company is 8.5% p.a. and A wishes to pay back the amount in 21 years. Compute the amount of loan which can be availed by him? (a) 450000 (b) 442356 (c) 415376 (d) 415672

Financial Planning Academy

83

18 If the effective rate of interest is 17.87% then on a debt that has quarterly payments, what is the nominal annual rate? (a) 16.78% (b) 18.92% (c) 20.93% (d) 21.00% (e) 22.36% 19 If a share of a stock provided a 19.5% nominal rate of return while the real rate of return was 14%, then the inflation rate was (a) 4.83% (b) 5.5% (c) 5.1% (d) 7.18% 20 What real rate of return is earned by a one year investor in a bond that was purchased for Rs.1000, has a 12% coupon and was sold for 980 when the inflation rate was 6% (a) 6.00% (b) 5.66% (c) 5.86% (d) 6.5% 21 The IDBI deep discount bond offers investors Rs.320000 after 20 years, for an initial investment of Rs.9500. The interest rate implied in the offer is: (a) 19.22% (b) 18.85% (c) 19.12% (d) 18.95% 22 M/S D Ltd has placed a deposit of Rs.5000 with a company at 15% p.a. interest being compounded semi-annually. In three years her investment will grow to: (a) 7250 (b) 7344 (c) 7500 (d) 7604 (e) 7716 23 How much should a company invest at the beginning of each year at 14% so that it can redeem debentures of Rs.10 Lakh at the end of year 10 (a) 45363 (b) 48195 (c) 51714 (d) 65236

Financial Planning Academy

84

24 A risk free stock has a beta of (a) -1 (b) Zero (c) 0.5 (d) 1 25 Choose the one which gives the highest return among the following (assume interest @14% p.a.) (a) Rs.100000 now (b) Rs.200000 after 6 years (c) Rs.15000 p.a. in perpetuity (d) Rs.1000 per month for a year and Rs.95000 at the end of the year (e) Rs.18000 per year for the next 10 years 26 If Rs.100000 becomes Rs.150000 in 2.5 years when compounding is done semi annually, find the annualized rate of return? (a) 16.89% (b) 16.50% (c) 15.95% (d) 16.00% 27 An amount becomes Rs.562778 in 5 years when for the first 4 years rate of interest is 6% p.a. compounded quarterly and in the 5th year compounding is done annually. Find the amount invested? (a) 430000 (b) 418484 (c) 418383 (d) 400000 28 Compute the present value of an amount which becomes Rs.464202 in 5 years when for the first 3 years compounding is annually and rate of interest is 9% and for the next 2 years compounding is done quarterly. (a) 290000 (b) 300000 (c) 310000 (d) 298000 29 Compute the present value of an annuity, which pays Rs.2000 per annum for 10 years when interest rate for the first 5 years is 5% and for the next 5 years is 8%. (a) 15849 (b) 14379 (c) 14916 (d) 15310

Financial Planning Academy

85

30 Total risk is measured by: (a) Beta (b) Standard deviation (c) Variance (d) Any of these 31 A companys relative capital structure can be known from its (a) Debt- equity ratios (b) P/E ratios (c) Liquidity ratios (d) None of these 32 The Indian taxation system is: (a) Progressive (b) Regressive (c) Assertive (d) None of these (progressive means increase in tax rates with increase in income) 33 The CFP certificant will not indulge in any practices which are detrimental to the profession is a requirement of which code of ethics? (a) The code of ethics of fairness (b) The code of ethics of professionalism (c) The code of ethics of integrity (d) The code of ethics of compliance 34 Which of the following is true? (a) YTM is dependent on both interest income as well as capital gains (b) The closer to maturity, the less sensitive a bond is to interest rate risk (c) Callable debt is a debt that may be paid off early (d) All of the above are true 35 Why do firms issue debt? (a) Managers like to have debt on the balance sheet as it gives them more discretion as to investment than does equity (b) Debt has lower cost than equity (c) Debt decreases the financial leverage of the firm (d) None of these

Financial Planning Academy

86

36 On your lease you pay Rs.500 at the end of each month for the next 12 months. Now your landlord offers you a new 1-year lease, which calls for zero rent for 3 months, then rental payments of pRs.700 at the end of each month for the next 9 months. You keep your money in a bank time deposit that pays a nominal annual rate of 5 %. By what amount would your net worth change if you accept the new lease? (a) Rs.509.81 (b) Rs.253.62 (c) Rs.125.30 (d) Rs.253.62 (e) Rs.509.81 37 Four years ago ABC Corporation issued 10 years bonds with a coupon rate of 11%.If the investors required rate of return (IRR) on this bond is currently 9%, the bonds will be priced: (a) Below par (b) Above par (c) Above and below par (d) At par 38 Returns Standard deviation Risk free return 8% Calculate the Sharpe index of performance for the fund over the evaluation period. (a) 0.3913 (b) 0.4286 (c) 0.4783 (d) 0.5238 (e) 0.5870 39 The standard deviation of the returns of a portfolio of securities will be ----------- the weighted average of the standard deviation of returns of the individual component securities. (a) Equal to (b) Less than (c) Greater than (d) Less than or equal to (depending on the correlation between securities) Performance fund 19% 23% Benchmark index 17% 21%

Financial Planning Academy

87

40 You wish to save for your sons education the present cost of which is Rs.320000 and is expected to increase by 6% every year. If your son is 12 years old and will require money in 8 years time, what is the amount of investment to be made if it is likely to earn 12% rate of return? (a) 205993 (b) 205670 (c) 210000 (d) 209553 41 The investment portfolio for a defined benefit plan has declined in value during a year in which most financial market investments have incurred losses. Which one of the following entities would be impacted most by this decline in portfolio value? (a) Individuals participating in the plan (b) Company sponsoring the plan (c) Investment banker handling the plan (d) Plan underwriters 42 Which of the following would result in the largest increase in the price of a diversified common stock mutual fund? (a) Unexpected inflation (b) Expected dividend increase (c) Unexpected corporate earnings growth (d) Expected increase in the prime interest rate 43 Sudha has invested Rs.12000 for 8 years at the rate of interest of 6%.What amount she will get after 8 years if amount is compounding annually for first 5 years and semi-annually for last 3 years. (a) 19803.43 (b) 19174.92 (c) 18976.24 (d) 19203.46 44 In the above question if in the first 5 years compounding is annually and next 2 years compounding is quarterly and in the last 1 year compounding is monthly. Calculate the amount of money accumulated by Sudha in 8 years. (a) 19205.75 (b) 19706.50 (c) 20000.89 (d) 19887.43

Financial Planning Academy

88

45 Ramesh will receive Rs.25000 & Rs.15000 at the end of 14 &15 year respectively. If rate of return is 6%.Compute the present value of the amount? (a) 17316.49 (b) 17500.50 (c) 18200.00 (d) 17432.88 46 The 8% RBI Bonds cannot be invested by following category of investors? (a) Scientific Research Organization (b) Companies under Companies Act (c) NRI (d) Trusts 47 What is the effective annual rate if the stated nominal rate is 12% per annum compounded monthly? (a) 12.55% (b) 12.36% (c) 12.68% (d) 12.49% 48 In the event of a loss due to an insured event the principle of indemnity ensures that (a) The compensation paid to the insured is always less than the loss (b) The compensation is equal to the loss (c) The compensation paid is more than the loss 49 In fire insurance pro rata means (a) Rating as per occupancy (b) Proportionate (c) Condition of average (d) None of them 50 Miss Rima works at the post office. She deposits Rs.1000 on every birthday into a retirement plan which paid an interest rate of 8% from the age of 20 years until she retired. If she had Rs.237941.22 in her retirement plan when she retired, at what age did she retire? (a) 55 (b) 60 (c) 59 (d) 57

Financial Planning Academy

89

51 Anil has purchased units of mutual fund on 15.03 1999 for Rs.15000 and sold on 12.04.2001. If cost of inflation index for 1998-99, 1999-00,2000-01 are 351,389 & 406 respectively. Compute cost inflation adjustment factor and calculate the indexed cost of acquisition? (a) 1.1567 &17350.43 (b) 1.1345 & 17650.34 (c) 1.1597 & 17890.55 (d) 1.1645 & 16745.23 52 Radha has received interest on Govt. securities Rs.6000, Dividend from mutual funds Rs.3000, Interest on RBI Bonds Rs.6000. Compute the amount of deduction he will get under Sec 80L. (a) Rs.15000 (b) Rs.12000 (c) Rs.10000 (d) Nil ( Sec 80L is discontinued.) 53 Sudha has to incur Rs.10000 for his household expenses,Rs.15000 for payment of mortgage loan and Rs.5000 for maintenance of car per month. He has Rs.150000 in Bank account, Equity share of Rs.900000 and house property of Rs.150000.Compute his liquidity ratio? (a) 8 (b) 9.5 (c) 5 (d) 6 54 BSE sensex is ------------- index. (a) Price weighted (b) Market capitalization (c) Market capitalization X price (d) None of these 55 Post office monthly income can be saved through (a) Cheque (b) Cash (c) Draft (d) All of the above 56 Person who declare will after the death of a person is called (a) Testator (b) Executor (c) Testate (d) Deceased

Financial Planning Academy

90

57 A person is about to be retired. He has taken adequate insurance. He has sufficient income to incur his regular expenses. He has his wife only in the family. Then he should takes (a) Life cover (b) Property Protection (c) Health protection (d) Retirement planning 58 If a partnership firm is filing its income tax return timely then whether it is required audit its accounts (a) Yes (b) No 59 Testamentary Trust comes in the force of from the date of (a) Death (b) Retirement (c) Children become minor 60 Mr. Y has purchased a bond of Rs.1000 face value. Bonds are issued for duration of 10 years. Coupon rate of the bond is 10% and current price of the bond is Rs.850. Calculate YTM. (a) 12.735 (b) 10.85 (c) 11.56 (d) 8.025 61 Sita has purchased a bond of Rs.1000 face value, coupon rate is 10%. Bonds are issued for the duration of 10 years. YTM of the bond is 8.5%. Compute current price of the bond. (a) 1098 (b) 956 (c) 1256 (d) 1245 62 A member shall clearly disclose to all prospective clients the capacity in which they are able to provide financial planning services. The above rule comes under the following code of ethic of (a) Competence (b) Fairness (c) Confidentiality (d) Professionalism to

Financial Planning Academy

91

63 Which of the following will help us in stock analysis? (a) Standard deviation of stock (b) Beta of stock (c) Return from stock (d) All of the above 64 Fundamental analysis stands for (a) Selecting stocks on the basis of EPS (b) Selecting stocks on the basis of book value (c) Selecting stocks on the basis of market value (d) All of the above (e) None of the above 65 Technical Analysis Stands for (a) Timing the market (b) Analysing demand and supply of stock (c) Comparing book value and market value (d) A and B both (e) B and C both Attempt the following questions based on the information given as under Stock Standard Beta Average return deviation A 23% 1.0 18% B 25% 1.2 17% C 20% 0.9 16% Return on 18% 21% market security Risk free return 8%

66 Compute Sharpe ratio for security A (a) 0.4348 (b) 0.4536 (c) 10.200 (d) 8.333 67 Compute Treynor ratio for security A (a) 10.00 (b) 9.991 (c) 8.333 (d) 10.02

Financial Planning Academy

92

68 Compute Jensen ratio or Jensens alpha for security A (a) 3.00 (b) 2.87 (c) -3.00 (d) 2.98 69 Compute Sharpe ratio for Security B (a) 0.398 (b) 0.360 (c) 0.345 (d) 0.456 70 Compute Treynor ratio for security B (a) 7.89 (b) 7.50 (c) 6.98 (d) 7.02 71 Compute Jensen s alpha for security B (a) -6.60 (b) 6.60 (c) 5.79 (d) 6.12 72 Compute Sharpe ratio for security C (a) 0.45 (b) 0.39 (c) 0.40 (d) 0.49 73 Compute Treynor ratio for security C (a) 8.89 (b) 8.80 (c) 8.75 (d) 8.56 74 Compute Jensen ratio for security C (a) 3.70 (b) 3.67 (c) -3.70 (d) 3.56

Financial Planning Academy

93

75 Compute Sharpe ratio for market security (a) 0.752 (b) 0.722 (c) 0.768 (d) 0.678 76 Standard deviation is (a) Measure of return (b) Measure of risk (c) Measure of covariance (d) All of the above 77 Standard deviation calculations require (a) Expected Return (b) Actual Return (c) Deviation between actual return and expected return (d) All of the above (e) None of the above 78 What is a Repo? (a) Repo is repurchase agreement (b) Repo involves simultaneous sale and repurchase (c) The difference between the sale and repurchase price represents cost of carry (d) All of the above 79 You can earn a return of 15% by investing in equity shares on your own. You are considering a recently announced equity mutual fund scheme where the initial issue expenses are 5% and the recurring expenses are expected to be 2%. How much should the mutual fund scheme earn to provide a return of 15% to you? (a) 17.00% (b) 17.75% (c) 17.79% (d) 18.25% (Hint: Out of Rs.100 invested by a investor only Rs.95 will be invested by Mutual Fund and has to generate return of 15% after deducting 2% recurring expenses) 80 What is a Subsidiary General Account? (a) This facility is available to large banks and financial institutions. (b) It is a facility to hold investment in G-Sec and T-Bills in the electronic form (c) Trade is settled through delivery verses payment system (d) All of the above are features of SGL account

Financial Planning Academy

94

81 What is the present value of Rs.10000 receivable after 6 years hence if the rate of discount is 10%. (a) 5465 (b) 5675 (c) 5645 (d) 6000 82 Suppose you expect to receive Rs15000 annually for 5 years, each receipt occurring at the end of the year. What is the present value of this stream of benefits if the discount rate is 10%? (a) -56861.80 (b) -57463.75 (c) -75000.00 (d) 55000.00 83 You want to take a trip to US which costs Rs.1000000 the cost is expected to remain unchanged in nominal terms. You can save annually Rs.50000 to fulfill your desire. How long will you have to wait if your savings earn an interest rate of 12%. (a) 10.79 years (b) 11.00 years (c) 11.23 years (d) 10.99 years 84 Suppose a firm borrows Rs.1000000 at an interest rate of 13%.The loan is to be paid in 5 equal installments payable at the end of each of the next 5 years. The annual installment payable will be how much? (a) 284314 (b) 284390 (c) 285436 (d) 285467 85 Your father deposits Rs.500000 on retirement in a bank which pays 10% annual interest compounded semi annually. How much can be withdrawn every year for a period of 10 years? (a) 80242.58 (b) 80534.20 (c) 82345.15 (d) 40121.29 86 If you invest Rs.5000 today at a compound interest rate of 9%, what will be its future value after 80 years? (a) 4932758.34 (b) 3205685.10 (c) 5000000.00 (d) None of these

Financial Planning Academy

95

87 A 10 year, 12% coupon bond has a par value of Rs.10000.The required yield on this bond is 14%. Compute the price of the bond. (a) 8900.78 (b) 8956.78 (c) 9012.34 (d) 8876.77 88 In the same question find the price of the bond if interest is compounded semi-annually? (a) 8940.59 (b) 8765.25 (c) 8956.78 (d) 8900.45 89 A 10 year 10% coupon bond has a par value of Rs.1000 and number of years remaining to maturity are 8. Similar bonds in the market are traded at 9% and compounding of interest is quarterly. Find the value of bond at which it can be sold? (a) 1056.59 (b) 1066.45 (c) 1059.89 (d) 1089.59 90 In the same question if compounding is monthly, calculate the price of the bond? (a) 1056.65 (b) 1089.67 (c) 1076.45 (d) 1056.99 91 The market price of a Rs.1000 par value bond carrying a coupon rate of 14% and maturing after 5 years is Rs.1050. What is the yield to maturity on this bond? (a) 12.60% (b) 12.35% (c) 12.78% (d) 12.98% 92 The equity stock of X Ltd is currently valued at Rs.30 per share. The dividend expected next year is Rs.2.00. The investors required rate of return is on this stock is 15%.If the company will grow at constant rate, what is the expected growth rate? (a) 9.2% (b) 8.3% (c) 8.9% (d) 10% (P0=D1/r-g)

Financial Planning Academy

96

93 The expected dividend per share on the equity share of A Ltd is Rs.2.00.(D1) The growth rate of dividend is 5%. Calculate the intrinsic value of the share if the required rate of return is 15%? (a) Rs.20.00 (b) Rs.21.00 (c) Rs.20.75 (d) Rs.22.00 (P0= D1/r-g, D1 is expected dividend next year, g is growth rate and r is required return) 94 What is At The Money option in case of call and put options? (a) In both the options when Exercise price is equal to Market price ( spot price) (b) When Exercise price >Spot price (c) When Exercise price<Spot price (d) All of the above 95 What is In the Money option in case of call option? (a) When X<Spot Price (b) When X > Spot Price 96 What is In the Money option in case of put option? (a) When X>Spot price (b) When X< Spot price 97 What is Out of The money option in case of call option? (a) When X>Spot price (b) When X < Spot price 98 What is Out of the Money in put option? (a) When X < Spot price (b) When X>Spot price 99 Strategic asset allocations refers to the long term normal asset mix sought by the investor (a) Yes (b) No 100 Tactical asset allocations involve a conscious departure from normal asset mix to enhance the performance of fund. (a) Yes (b) No (c) It is something else

Financial Planning Academy

97

101. If a client needs to accumulate wealth but is risk-averse, which of the following is the most crucial action the planner needs to take to have the client achieve the goal of wealth accumulation? Advise investing the clients assets: a) In the products which will bring the highest return to the client regardless of risk b) In products which produce high income for the client because fixed income products are generally safe c) In diversified mutual funds because of the protection which diversity provides d) After determining the clients risk tolerance 102. John estimates his opportunity cost on investment at 10.5% compounded annually. Which one of the following is the best investment opportunity for John? a) To receive 45,000 today b) To receive 120,000 at the end of 10 years c) To receive 5,500 at the beginning of each year for 15 years. d) To receive 5,500 at the end of each year for 19 years 103. A client provides a current personal balance sheet to the financial planner during the initial data gathering phase of the financial planning process. This financial statement will enable the financial planner to gain an understanding of all of the following except the: a) Diversification of the clients assets b) Size of the clients net cash flow c) Clients liquidity position d) Clients use of debt 104. Which best describes high unemployment, with low interest rates? a) Peak b) Recession c) Trough d) Expansion 105. Your client has some ABC shares. He asked you to calculate the dividend yield for an ABC share. ABC Company distributed 500,000 of profits last year. Earnings were 800,000. The market price of a share is currently 5.00, and there are 1 million shares issued. What is ABCs dividend yield? a) 6.25 percent b) 10.0 percent c) 16.7 percent d) None of the above

Financial Planning Academy

98

106. Which of the following are non-diversifiable risks? (1) Business risk (2) Management risk (3) Company or industry risk (4) Market risk (5) Interest rate risk (6) Purchasing power risk a) (4), (5) and (6) only b) (1), (2) and (3) only c) (5), (6) and (2) only d) (1), (3) and (4) only 107. A client purchased a zero coupon bond 6.5 years ago for 525. If the bond matures today and the face value is 1,000, what is the average annual compound rate of return (calculated semi-annually) that the client realised on her investment? a) 11.3372% b) 10.5713% c) 10.4000% d) 10.1630% 108. Which of the following statement is correct? a) Insurance is a dispersion of actual from expected results b) A hazard is the cause of financial loss c) Speculative risk involves the chance of loss or no loss d) Insurance is a device for reducing risk by having a large pool of people share in the financial loss suffered by members of the pool 109. In advising a client on his retirement needs, which of the following statements best describes the replacement ratio method: a) It is a longer-term projection b) Takes into account likely changes in income and expenses, and tries to determine the required savings to meet his or her retirement goals c) Focuses on projected expenses in retirement d) Assumes the standard of living just prior to retirement will determine the standard of living during retirement 110. Your client is unsure of the meaning of the term codicil. A codicil is: a) A document used to make an alteration to a will b) The statement made at the end of a will that it has been duly attested c) A term used to cover grants of probate to the legal personal representative d) A gift of land or real estate in a will

Financial Planning Academy

99

You might also like